Метод обратной матрицы метод гаусса: Нахождение обратной матрицы методом Гаусса

{-1}$ не существует.

Когда мы придём к последней строке, матрица до черты станет верхней треугольной, т.е. все элементы под главной диагональю будут равны нулю. Это будет означать конец прямого хода метода Гаусса.

Содержание

Обратный ход метода Гаусса

На этом этапе мы поднимаемся по матрице «снизу вверх». Сначала используем последнюю строку $r_n$, затем предпоследнюю $r_{n-1}$ и так далее, пока не дойдём до первой строки. С каждой строкой выполняем однотипные операции.

Пусть, например, речь идёт о некоей k-й строке. Матрица, расположенная до черты, содержит в строке $r_k$ диагональный элемент $a_{kk}$. Если $a_{kk}=1$, то это нас вполне устраивает, а если $a_{kk}\neq{1}$, то просто умножаем строку $r_k$ на коэффициент $\frac{1}{a_{kk}}$, чтобы диагональный элемент стал равен 1. Затем с помощью строки $r_k$ обнуляем элементы k-го столбца, расположенные над строкой $r_k$.

Как только мы дойдём до первой строки, матрица до черты станет единичной, и алгоритм завершится.{-1}\right)$.

Метод Гаусса

Прямой ход метода Гаусса

Первый шаг

На первом шаге прямого хода мы работаем с первой строкой. Первый элемент этой строки (число -5) не равен нулю, поэтому можем приступать к обнулению ненулевых элементов первого столбца, расположенных под первой строкой. Однако для тех преобразований, которые мы станем делать для обнуления элементов, удобно, когда ведущий элемент используемой строки равен 1 или -1. Почему это так, станет ясно из дальнейших действий. Чтобы ведущий элемент текущей строки стал равен -1, поменяем местами первую строку с одной из нижележащих строк – с второй строкой:

$$ \left(\begin{array}{ccc|ccc} -5 & 23 & -24 & 1 & 0 & 0\\ -1 & 4 & -5 & 0 & 1 & 0\\ 9 & -40 & 43 & 0 & 0 & 1 \end{array}\right) \overset{r_1\leftrightarrow{r_2}}{\rightarrow} \left(\begin{array}{ccc|ccc} \boldred{-1} & 4 & -5 & 0 & 1 & 0\\ \normblue{-5} & 23 & -24 & 1 & 0 & 0\\ \normblue{9} & -40 & 43 & 0 & 0 & 1 \end{array}\right) $$

Теперь ведущий элемент первой строки стал равен -1 (я выделил этот элемент красным цветом). Приступим к обнулению ненулевых элементов первого столбца, лежащих под первой строкой (они выделены синим цветом). Для этого над строками матрицы нужно выполнить такие действия:

$$ \begin{aligned} &r_2-\frac{\normblue{-5}}{\boldred{-1}}\cdot{r_1}=r_2-5r_1;\\ &r_3-\frac{\normblue{9}}{\boldred{-1}}\cdot{r_1}=r_3+9r_1. \end{aligned} $$

Запись $r_2-5r_1$ означает, что от элементов второй строки вычли соответствующие элементы первой строки, умноженные на пять. Результат записывают на место второй строки в новую матрицу. Если с устным выполнением такой операции возникают сложности, то это действие можно выполнить отдельно:

$$ r_2-5r_1 =(-5;\;23;\;-24;\;1;\;0;\;0)-5\cdot(-1;\;4;\;-5;\;0;\;1;\;0)=\\ =(-5;\;23;\;-24;\;1;\;0;\;0)-(-5;\;20;\;-25;\;0;\;5;\;0) =(0;\;3;\;1;\;1;\;-5;\;0) $$

Действие $r_3+9r_1$ выполняется аналогично. Первую строку мы не трогали, поэтому в новую матрицу она перейдёт без изменений:

$$ \left(\begin{array}{ccc|ccc} -1 & 4 & -5 & 0 & 1 & 0\\ -5 & 23 & -24 & 1 & 0 & 0\\ 9 & -40 & 43 & 0 & 0 & 1 \end{array}\right) \begin{array} {l} \phantom{0}\\ r_2-5r_1 \\ r_3+9r_1 \end{array} \rightarrow \left(\begin{array}{ccc|ccc} -1 & 4 & -5 & 0 & 1 & 0\\ 0 & 3 & 1 & 1 & -5 & 0\\ 0 & -4 & -2 & 0 & 9 & 1 \end{array}\right) $$

На этом первый шаг закончен. Нулевых строк в матрице до черты не возникло, поэтому продолжаем решение. Кстати, теперь, я полагаю, ясно, зачем надо было менять местами строки. Если бы не смена мест строк, нам пришлось бы выполнять действия $r_2-\frac{1}{5}\cdot{r_1}$ и $r_3+\frac{9}{5}\cdot{r_1}$, что привело бы к появлению дробей. А легче, разумеется, работать с целыми числами, чем с дробями.

Второй шаг

На втором шаге прямого хода мы работаем с второй строкой. Второй элемент этой строки (число 3) не равен нулю, поэтому можем приступать к обнулению ненулевых элементов второго столбца, расположенных под второй строкой:

$$ \left(\begin{array}{ccc|ccc} -1 & 4 & -5 & 0 & 1 & 0\\ 0 & 3 & 1 & 1 & -5 & 0\\ 0 & -4 & -2 & 0 & 9 & 1 \end{array}\right) \begin{array} {l} \phantom{0}\\ \phantom{0} \\ r_3+4/3\cdot{r_2} \end{array} \rightarrow \left(\begin{array}{ccc|ccc} -1 & 4 & -5 & 0 & 1 & 0\\ 0 & 3 & 1 & 1 & -5 & 0\\ 0 & 0 & -2/3 & 4/3 & 7/3 & 1 \end{array}\right) $$

Матрица до черты стала верхней треугольной, поэтому прямой ход метода Гаусса окончен.

Пару слов насчёт действий со строками, которые мы выполняли на втором шаге. На первом шаге мы меняли местами строки, чтобы ведущий элемент первой строки стал равен -1. Здесь такая смена строк ничего не даст, так как доступна к обмену лишь третья строка, а у неё ведущий элемент тоже не равен ни 1, ни -1. В этом случае можно выполнить дополнительное преобразование со второй строкой: $r_2+r_3$:

$$ \left(\begin{array}{ccc|ccc} -1 & 4 & -5 & 0 & 1 & 0\\ 0 & 3 & 1 & 1 & -5 & 0\\ 0 & -4 & -2 & 0 & 9 & 1 \end{array}\right) \begin{array} {l} \phantom{0}\\ r_2+r_3 \\ \phantom{0} \end{array} \rightarrow \left(\begin{array}{ccc|ccc} -1 & 4 & -5 & 0 & 1 & 0\\ 0 & -1 & -1 & 1 & 4 & 1\\ 0 & -4 & -2 & 0 & 9 & 1 \end{array}\right) $$

После этого текущий шаг прямого хода будет продолжен без дробей. Можно было сделать и такое действие: $3r_3+4r_2$, тогда и необходимый элемент третьего столбца был бы обнулён, и дробей бы не появилось. Выполнять такие действия или нет – надо смотреть по ситуации. Если работы с дробями предвидится немного, то особого смысла в попытках их избежать нет. Если же нас ожидают ещё несколько шагов прямого хода, то, возможно, лучше упростить себе расчёты и выполнить вспомогательное действие, чтобы потом не работать с дробями. К слову, если есть необходимость избавиться от дробей в некоей строке, то можно просто домножить данную строку на соответствующий коэффициент. Например, строку $\left(\frac{1}{3};\;-\frac{4}{5};\;2;0\right)$ можно домножить на число 15, тогда дроби исчезнут, и строка станет такой: $\left(5;\;-12;\;30;0\right)$.

Обратный ход метода Гаусса

Первый шаг

На первом шаге обратного хода мы работаем с последней, т.е. третьей строкой матрицы. Посмотрим на диагональный элемент в третьей строке: он равен $-\frac{2}{3}$. Сделаем этот элемент единицей, домножив третью строку на $-\frac{3}{2}$, а затем с помощью третьей строки обнулим ненулевые элементы третьего столбца, расположенные над третьей строкой:

$$ \left(\begin{array}{ccc|ccc} -1 & 4 & -5 & 0 & 1 & 0\\ 0 & 3 & 1 & 1 & -5 & 0\\ 0 & 0 & -2/3 & 4/3 & 7/3 & 1 \end{array}\right) \begin{array} {l} \phantom{0}\\ \phantom{0}\\ -3/2\cdot{r_3} \end{array} \rightarrow\\ \left(\begin{array}{ccc|ccc} -1 & 4 & -5 & 0 & 1 & 0\\ 0 & 3 & 1 & 1 & -5 & 0\\ 0 & 0 & 1 & -2 & -7/2 & -3/2 \end{array}\right) \begin{array} {l} r_1+5r_3 \phantom{0}\\ r_2-r_3\\ \phantom{0} \end{array} \rightarrow \left(\begin{array}{ccc|ccc} -1 & 4 & 0 & -10 & -33/2 & -15/2\\ 0 & 3 & 0 & 3 & -3/2 & 3/2\\ 0 & 0 & 1 & -2 & -7/2 & -3/2 \end{array}\right) $$

Второй шаг

На втором шаге обратного хода мы работаем с предпоследней, т.е. второй строкой матрицы. Посмотрим на диагональный элемент во второй строке: он равен 3. Сделаем этот элемент единицей, домножив вторую строку на $\frac{1}{3}$, а затем с помощью второй строки обнулим ненулевой элемент второго столбца, расположенный над второй строкой:

$$ \left(\begin{array}{ccc|ccc} -1 & 4 & 0 & -10 & -33/2 & -15/2\\ 0 & 3 & 0 & 3 & -3/2 & 3/2\\ 0 & 0 & 1 & -2 & -7/2 & -3/2 \end{array}\right) \begin{array} {l} \phantom{0}\\ 1/3\cdot{r_2} \\ \phantom{0} \end{array} \rightarrow\\ \left(\begin{array}{ccc|ccc} -1 & 4 & 0 & -10 & -33/2 & -15/2\\ 0 & 1 & 0 & 1 & -1/2 & 1/2\\ 0 & 0 & 1 & -2 & -7/2 & -3/2 \end{array}\right) \begin{array} {l} r_1-4r_2\\ \phantom{0} \\ \phantom{0} \end{array} \rightarrow \left(\begin{array}{ccc|ccc} -1 & 0 & 0 & -14 & -29/2 & -19/2\\ 0 & 1 & 0 & 1 & -1/2 & 1/2\\ 0 & 0 & 1 & -2 & -7/2 & -3/2 \end{array}\right) $$

Третий шаг

Работаем с первой строкой.{-1} =\left(\begin{array}{ccc} 14 & 29/2 & 19/2\\ 1 & -1/2 & 1/2\\ -2 & -7/2 & -3/2 \end{array}\right) $$

Если пропустить все пояснения, то решение будет таким:

$$ \left(\begin{array}{ccc|ccc} -5 & 23 & -24 & 1 & 0 & 0\\ -1 & 4 & -5 & 0 & 1 & 0\\ 9 & -40 & 43 & 0 & 0 & 1 \end{array}\right) \overset{r_1\leftrightarrow{r_2}}{\rightarrow} $$ $$ \rightarrow\left(\begin{array}{ccc|ccc} -1 & 4 & -5 & 0 & 1 & 0\\ -5 & 23 & -24 & 1 & 0 & 0\\ 9 & -40 & 43 & 0 & 0 & 1 \end{array}\right) \begin{array} {l} \phantom{0}\\ r_2-5r_1 \\ r_3+9r_1 \end{array} \rightarrow \left(\begin{array}{ccc|ccc} -1 & 4 & -5 & 0 & 1 & 0\\ 0 & 3 & 1 & 1 & -5 & 0\\ 0 & -4 & -2 & 0 & 9 & 1 \end{array}\right) \begin{array} {l} \phantom{0}\\ \phantom{0} \\ r_3+4/3\cdot{r_2} \end{array} \rightarrow $$ $$ \rightarrow\left(\begin{array}{ccc|ccc} -1 & 4 & -5 & 0 & 1 & 0\\ 0 & 3 & 1 & 1 & -5 & 0\\ 0 & 0 & -2/3 & 4/3 & 7/3 & 1 \end{array}\right) \begin{array} {l} \phantom{0}\\ \phantom{0}\\ -3/2\cdot{r_3} \end{array} \rightarrow \left(\begin{array}{ccc|ccc} -1 & 4 & -5 & 0 & 1 & 0\\ 0 & 3 & 1 & 1 & -5 & 0\\ 0 & 0 & 1 & -2 & -7/2 & -3/2 \end{array}\right) \begin{array} {l} r_1+5r_3 \phantom{0}\\ r_2-r_3\\ \phantom{0} \end{array} \rightarrow $$ $$ \rightarrow\left(\begin{array}{ccc|ccc} -1 & 4 & 0 & -10 & -33/2 & -15/2\\ 0 & 3 & 0 & 3 & -3/2 & 3/2\\ 0 & 0 & 1 & -2 & -7/2 & -3/2 \end{array}\right) \begin{array} {l} \phantom{0}\\ 1/3\cdot{r_2} \\ \phantom{0} \end{array} \rightarrow \left(\begin{array}{ccc|ccc} -1 & 4 & 0 & -10 & -33/2 & -15/2\\ 0 & 1 & 0 & 1 & -1/2 & 1/2\\ 0 & 0 & 1 & -2 & -7/2 & -3/2 \end{array}\right) \begin{array} {l} r_1-4r_2\\ \phantom{0} \\ \phantom{0} \end{array} \rightarrow $$ $$ \rightarrow\left(\begin{array}{ccc|ccc} -1 & 0 & 0 & -14 & -29/2 & -19/2\\ 0 & 1 & 0 & 1 & -1/2 & 1/2\\ 0 & 0 & 1 & -2 & -7/2 & -3/2 \end{array}\right) \begin{array} {l} -1\cdot{r_1}\\ \phantom{0} \\ \phantom{0} \end{array} \rightarrow \left(\begin{array}{ccc|ccc} 1 & 0 & 0 & 14 & 29/2 & 19/2\\ 0 & 1 & 0 & 1 & -1/2 & 1/2\\ 0 & 0 & 1 & -2 & -7/2 & -3/2 \end{array}\right) $$

Теперь решим этот же пример методом Гаусса-Жордана.

Метод Гаусса-Жордана

Первый шаг

На первом шаге мы работаем с первой строкой. Первый элемент этой строки (число -5) не равен нулю, поэтому можем следовать стандартному алгоритму: домножить первую строку на $-\frac{1}{5}$, чтобы первый элемент стал равен единице, а затем обнулить все иные ненулевые элементы первого столбца. Однако, как и при решении методом Гаусса, удобно, когда ведущий элемент используемой строки равен 1 или -1. Поэтому как и на первом шаге метода Гаусса, поменяем местами первую строку с второй строкой:

$$ \left(\begin{array}{ccc|ccc} -5 & 23 & -24 & 1 & 0 & 0\\ -1 & 4 & -5 & 0 & 1 & 0\\ 9 & -40 & 43 & 0 & 0 & 1 \end{array}\right) \overset{r_1\leftrightarrow{r_2}}{\rightarrow} \left(\begin{array}{ccc|ccc} -1 & 4 & -5 & 0 & 1 & 0\\ \normblue{-5} & 23 & -24 & 1 & 0 & 0\\ \normblue{9} & -40 & 43 & 0 & 0 & 1 \end{array}\right) $$

Теперь первый элемент первой строки стал равен -1. Чтобы этот элемент стал равен 1, домножим первую строку на -1, а потом обнулим все остальные ненулевые элементы первого столбца (они выделены в матрице выше синим цветом):

$$ \left(\begin{array}{ccc|ccc} -1 & 4 & -5 & 0 & 1 & 0\\ -5 & 23 & -24 & 1 & 0 & 0\\ 9 & -40 & 43 & 0 & 0 & 1 \end{array}\right) \begin{array} {l} -1\cdot{r_1}\\ \phantom{0} \\ \phantom{0} \end{array} \rightarrow\\ \rightarrow\left(\begin{array}{ccc|ccc} 1 & -4 & 5 & 0 & -1 & 0\\ -5 & 23 & -24 & 1 & 0 & 0\\ 9 & -40 & 43 & 0 & 0 & 1 \end{array}\right) \begin{array} {l} \phantom{0}\\ r_2+5r_1 \\ r_3-9r_1 \end{array} \rightarrow \left(\begin{array}{ccc|ccc} 1 & -4 & 5 & 0 & -1 & 0\\ 0 & 3 & 1 & 1 & -5 & 0\\ 0 & -4 & -2 & 0 & 9 & 1 \end{array}\right) $$

На этом первый шаг закончен. Нулевых строк в матрице до черты не возникло, поэтому продолжаем решение.

Второй шаг

На втором шаге мы работаем с второй строкой. Второй элемент этой строки (число 3) не равен нулю, поэтому домножаем вторую строку на $\frac{1}{3}$, чтобы второй элемент стал равен единице, а затем обнуляем все иные ненулевые элементы второго столбца.

$$ \left(\begin{array}{ccc|ccc} 1 & -4 & 5 & 0 & -1 & 0\\ 0 & 3 & 1 & 1 & -5 & 0\\ 0 & -4 & -2 & 0 & 9 & 1 \end{array}\right) \begin{array} {l} \phantom{0}\\1/3\cdot{r_2} \\\phantom{0}\end{array} \rightarrow\\ \rightarrow\left(\begin{array}{ccc|ccc} 1 & -4 & 5 & 0 & -1 & 0\\ 0 & 1 & 1/3 & 1/3 & -5/3 & 0\\ 0 & -4 & -2 & 0 & 9 & 1 \end{array}\right) \begin{array} {l} r_1+4r_2\\ \phantom{0} \\ r_3+4r_2 \end{array} \rightarrow \left(\begin{array}{ccc|ccc} 1 & 0 & 19/3 & 4/3 & -23/3 & 0\\ 0 & 1 & 1/3 & 1/3 & -5/3 & 0\\ 0 & 0 & -2/3 & 4/3 & 7/3 & 1 \end{array}\right) $$

Замечание относительно облегчения работы с дробями, сделанное после второго шага прямого хода метода Гаусса, остаётся в силе и здесь.

Третий шаг

На третьем шаге мы работаем с третьей строкой. Третий элемент этой строки (число -2/3) не равен нулю, поэтому домножаем третью строку на $-\frac{3}{2}$, чтобы третий элемент стал равен единице, а затем обнуляем все иные ненулевые элементы третьего столбца.

$$ \left(\begin{array}{ccc|ccc} 1 & 0 & 19/3 & 4/3 & -23/3 & 0\\ 0 & 1 & 1/3 & 1/3 & -5/3 & 0\\ 0 & 0 & -2/3 & 4/3 & 7/3 & 1 \end{array}\right) \begin{array} {l} \phantom{0}\\\phantom{0} \\ -3/2\cdot{r_3}\end{array} \rightarrow\\ \rightarrow\left(\begin{array}{ccc|ccc} 1 & 0 & 19/3 & 4/3 & -23/3 & 0\\ 0 & 1 & 1/3 & 1/3 & -5/3 & 0\\ 0 & 0 & 1 & -2 & -7/2 & -3/2 \end{array}\right) \begin{array} {l} r_1-19/3r_3\\ r_2-1/3\cdot{r_3} \\ \phantom{0} \end{array} \rightarrow \left(\begin{array}{ccc|ccc} 1 & 0 & 0 & 14 & 29/2 & 19/2\\ 0 & 1 & 0 & 1 & -1/2 & 1/2\\ 0 & 0 & 1 & -2 & -7/2 & -3/2 \end{array}\right) $$

Матрица до черты стала единичной, преобразования завершены.{-1} =\left(\begin{array}{ccc} 14 & 29/2 & 19/2\\ 1 & -1/2 & 1/2\\ -2 & -7/2 & -3/2 \end{array}\right) $$

Если пропустить все пояснения, то решение будет таким:

$$ \left(\begin{array}{ccc|ccc} -5 & 23 & -24 & 1 & 0 & 0\\ -1 & 4 & -5 & 0 & 1 & 0\\ 9 & -40 & 43 & 0 & 0 & 1 \end{array}\right) \overset{r_1\leftrightarrow{r_2}}{\rightarrow} \left(\begin{array}{ccc|ccc} -1 & 4 & -5 & 0 & 1 & 0\\ -5 & 23 & -24 & 1 & 0 & 0\\ 9 & -40 & 43 & 0 & 0 & 1 \end{array}\right) \begin{array} {l} -1\cdot{r_1}\\ \phantom{0} \\ \phantom{0} \end{array} \rightarrow $$ $$ \rightarrow\left(\begin{array}{ccc|ccc} 1 & -4 & 5 & 0 & -1 & 0\\ -5 & 23 & -24 & 1 & 0 & 0\\ 9 & -40 & 43 & 0 & 0 & 1 \end{array}\right) \begin{array} {l} \phantom{0}\\ r_2+5r_1 \\ r_3-9r_1 \end{array} \rightarrow \left(\begin{array}{ccc|ccc} 1 & -4 & 5 & 0 & -1 & 0\\ 0 & 3 & 1 & 1 & -5 & 0\\ 0 & -4 & -2 & 0 & 9 & 1 \end{array}\right) \begin{array} {l} \phantom{0}\\1/3\cdot{r_2} \\\phantom{0}\end{array} \rightarrow $$ $$ \rightarrow\left(\begin{array}{ccc|ccc} 1 & -4 & 5 & 0 & -1 & 0\\ 0 & 1 & 1/3 & 1/3 & -5/3 & 0\\ 0 & -4 & -2 & 0 & 9 & 1 \end{array}\right) \begin{array} {l} r_1+4r_2\\ \phantom{0} \\ r_3+4r_2 \end{array} \rightarrow \left(\begin{array}{ccc|ccc} 1 & 0 & 19/3 & 4/3 & -23/3 & 0\\ 0 & 1 & 1/3 & 1/3 & -5/3 & 0\\ 0 & 0 & -2/3 & 4/3 & 7/3 & 1 \end{array}\right) \begin{array} {l} \phantom{0}\\\phantom{0} \\ -3/2\cdot{r_3}\end{array} \rightarrow $$ $$ \rightarrow\left(\begin{array}{ccc|ccc} 1 & 0 & 19/3 & 4/3 & -23/3 & 0\\ 0 & 1 & 1/3 & 1/3 & -5/3 & 0\\ 0 & 0 & 1 & -2 & -7/2 & -3/2 \end{array}\right) \begin{array} {l} r_1-19/3r_3\\ r_2-1/3\cdot{r_3} \\ \phantom{0} \end{array} \rightarrow \left(\begin{array}{ccc|ccc} 1 & 0 & 0 & 14 & 29/2 & 19/2\\ 0 & 1 & 0 & 1 & -1/2 & 1/2\\ 0 & 0 & 1 & -2 & -7/2 & -3/2 \end{array}\right) $$

Ответ: $A^{-1} =\left(\begin{array}{ccc} 14 & 29/2 & 19/2\\ 1 & -1/2 & 1/2\\ -2 & -7/2 & -3/2 \end{array}\right)$.{-1}$, если $A=\left(\begin{array}{cccc} -2 & 3 & 0 & 1\\ -6 & 9 & -2 & 7\\ 0 & -2 & -18 & 27\\ -4 & 5 & -8 & 14\end{array} \right)$.

Решение

В предыдущем примере были даны подробные пояснения каждого шага как метода Гаусса, так и метода Гаусса-Жордана. В этом примере я стану комментировать лишь некие нюансы, которые возникнут в ходе решения.

Метод Гаусса

$$ \left(\begin{array}{cccc|cccc} -2 & 3 & 0 & 1 & 1 & 0 & 0 & 0\\ -6 & 9 & -2 & 7 & 0 & 1 & 0 & 0 \\ 0 & -2 & -18 & 27 & 0 & 0 & 1 & 0\\ -4 & 5 & -8 & 14 & 0 & 0 & 0 & 1 \end{array} \right) \begin{array} {l} \phantom{0} \\ r_2-3r_1 \\ \phantom{0} \\ r_4-2r_1 \end{array} \rightarrow \left(\begin{array}{cccc|cccc} -2 & 3 & 0 & 1 & 1 & 0 & 0 & 0\\ 0 & 0 & -2 & 4 & -3 & 1 & 0 & 0 \\ 0 & -2 & -18 & 27 & 0 & 0 & 1 & 0\\ 0 & -1 & -8 & 12 & -2 & 0 & 0 & 1 \end{array} \right) $$

Пора переходить ко второму шагу прямого хода метода Гаусса. На этом шаге должна использоваться вторая строка, однако второй элемент данной строки равен нулю. Согласно алгоритму, нужно поменять местами вторую строку с одной из нижележащих строк, у которых второй элемент отличен от нуля. Поменяем местами вторую и четвёртую строки, а потом продолжим преобразования:

$$ \left(\begin{array}{cccc|cccc} -2 & 3 & 0 & 1 & 1 & 0 & 0 & 0\\ 0 & 0 & -2 & 4 & -3 & 1 & 0 & 0 \\ 0 & -2 & -18 & 27 & 0 & 0 & 1 & 0\\ 0 & -1 & -8 & 12 & -2 & 0 & 0 & 1 \end{array} \right) \overset{r_2\leftrightarrow{r_4}}{\rightarrow} \left(\begin{array}{cccc|cccc} -2 & 3 & 0 & 1 & 1 & 0 & 0 & 0\\ 0 & -1 & -8 & 12 & -2 & 0 & 0 & 1\\ 0 & -2 & -18 & 27 & 0 & 0 & 1 & 0\\ 0 & 0 & -2 & 4 & -3 & 1 & 0 & 0 \end{array} \right) \begin{array} {l} \phantom{0} \\ \phantom{0} \\ r_3-2r_2 \\ \phantom{0} \end{array} \rightarrow $$ $$ \rightarrow\left(\begin{array}{cccc|cccc} -2 & 3 & 0 & 1 & 1 & 0 & 0 & 0\\ 0 & -1 & -8 & 12 & -2 & 0 & 0 & 1\\ 0 & 0 & -2 & 3 & 4 & 0 & 1 & -2\\ 0 & 0 & -2 & 4 & -3 & 1 & 0 & 0 \end{array} \right) \begin{array} {l} \phantom{0} \\ \phantom{0} \\ \phantom{0} \\ r_4-r_3 \end{array} \rightarrow $$ $$ \rightarrow\left(\begin{array}{cccc|cccc} -2 & 3 & 0 & 1 & 1 & 0 & 0 & 0\\ 0 & -1 & -8 & 12 & -2 & 0 & 0 & 1\\ 0 & 0 & -2 & 3 & 4 & 0 & 1 & -2\\ 0 & 0 & 0 & 1 & -7 & 1 & -1 & 2 \end{array} \right) \begin{array} {l} r_1-r_4 \\ r_2-12r_4 \\ r_3-3r_1 \\ \phantom{0} \end{array} \rightarrow $$ $$ \rightarrow\left(\begin{array}{cccc|cccc} -2 & 3 & 0 & 0 & 8 & -1 & 1 & -2\\ 0 & -1 & -8 & 0 & 82 & -12 & 12 & -23\\ 0 & 0 & -2 & 0 & 25 & -3 & 4 & -8\\ 0 & 0 & 0 & 1 & -7 & 1 & -1 & 2 \end{array} \right) \begin{array} {l} \phantom{0} \\ \phantom{0} \\ -1/2\cdot{r_3} \\ \phantom{0} \end{array} \rightarrow $$ $$ \rightarrow\left(\begin{array}{cccc|cccc} -2 & 3 & 0 & 0 & 8 & -1 & 1 & -2\\ 0 & -1 & -8 & 0 & 82 & -12 & 12 & -23\\ 0 & 0 & 1 & 0 & -25/2 & 3/2 & -2 & 4\\ 0 & 0 & 0 & 1 & -7 & 1 & -1 & 2 \end{array} \right) \begin{array} {l} \phantom{0} \\ r_2+8r_3 \\ \phantom{0} \\ \phantom{0} \end{array} \rightarrow $$ $$ \rightarrow\left(\begin{array}{cccc|cccc} -2 & 3 & 0 & 0 & 8 & -1 & 1 & -2\\ 0 & -1 & 0 & 0 & -18 & 0 & -4 & 9\\ 0 & 0 & 1 & 0 & -25/2 & 3/2 & -2 & 4\\ 0 & 0 & 0 & 1 & -7 & 1 & -1 & 2 \end{array} \right) \begin{array} {l} \phantom{0} \\ -1\cdot{r_2} \\ \phantom{0} \\ \phantom{0} \end{array} \rightarrow $$ $$ \rightarrow\left(\begin{array}{cccc|cccc} -2 & 3 & 0 & 0 & 8 & -1 & 1 & -2\\ 0 & 1 & 0 & 0 & 18 & 0 & 4 & -9\\ 0 & 0 & 1 & 0 & -25/2 & 3/2 & -2 & 4\\ 0 & 0 & 0 & 1 & -7 & 1 & -1 & 2 \end{array} \right) \begin{array} {l} r_1-3r_2 \\ \phantom{0} \\ \phantom{0} \\ \phantom{0} \end{array} \rightarrow $$ $$ \rightarrow\left(\begin{array}{cccc|cccc} -2 & 0 & 0 & 0 & -46 & -1 & -11 & 25\\ 0 & 1 & 0 & 0 & 18 & 0 & 4 & -9\\ 0 & 0 & 1 & 0 & -25/2 & 3/2 & -2 & 4\\ 0 & 0 & 0 & 1 & -7 & 1 & -1 & 2 \end{array} \right) \begin{array} {l} -1/2\cdot{r_1} \\ \phantom{0} \\ \phantom{0} \\ \phantom{0} \end{array} \rightarrow $$ $$ \rightarrow\left(\begin{array}{cccc|cccc} 1 & 0 & 0 & 0 & 23 & 1/2 & 11/2 & -25/2\\ 0 & 1 & 0 & 0 & 18 & 0 & 4 & -9\\ 0 & 0 & 1 & 0 & -25/2 & 3/2 & -2 & 4\\ 0 & 0 & 0 & 1 & -7 & 1 & -1 & 2 \end{array}\right) $$

Из последней матрицы получаем ответ:

$$ A^{-1} =\left(\begin{array}{cccc} 23 & 1/2 & 11/2 & -25/2\\ 18 & 0 & 4 & -9\\ -25/2 & 3/2 & -2 & 4\\ -7 & 1 & -1 & 2 \end{array}\right) $$

Метод Гаусса-Жордана

$$ \left(\begin{array}{cccc|cccc} -2 & 3 & 0 & 1 & 1 & 0 & 0 & 0\\ -6 & 9 & -2 & 7 & 0 & 1 & 0 & 0 \\ 0 & -2 & -18 & 27 & 0 & 0 & 1 & 0\\ -4 & 5 & -8 & 14 & 0 & 0 & 0 & 1 \end{array} \right) \begin{array} {l} -1/2\cdot{r_1} \\ \phantom{0} \\ \phantom{0} \\ \phantom{0} \end{array} \rightarrow $$ $$ \rightarrow\left(\begin{array}{cccc|cccc} 1 & -3/2 & 0 & -1/2 & -1/2 & 0 & 0 & 0\\ -6 & 9 & -2 & 7 & 0 & 1 & 0 & 0 \\ 0 & -2 & -18 & 27 & 0 & 0 & 1 & 0\\ -4 & 5 & -8 & 14 & 0 & 0 & 0 & 1 \end{array} \right) \begin{array} {l} \phantom{0} \\ r_2+6r_1 \\ \phantom{0} \\ r_4+4r_1 \end{array} \rightarrow $$ $$ \rightarrow\left(\begin{array}{cccc|cccc} 1 & -3/2 & 0 & -1/2 & -1/2 & 0 & 0 & 0\\ 0 & 0 & -2 & 4 & -3 & 1 & 0 & 0 \\ 0 & -2 & -18 & 27 & 0 & 0 & 1 & 0\\ 0 & -1 & -8 & 12 & -2 & 0 & 0 & 1 \end{array} \right) $$

Пора переходить ко второму шагу метода Гаусса-Жордана. На этом шаге должна использоваться вторая строка, однако второй элемент данной строки равен нулю. Согласно алгоритму, нужно поменять местами вторую строку с одной из нижележащих строк, у которых второй элемент отличен от нуля. Поменяем местами вторую и четвёртую строки, а потом продолжим преобразования:

$$ \left(\begin{array}{cccc|cccc} 1 & -3/2 & 0 & -1/2 & -1/2 & 0 & 0 & 0\\ 0 & 0 & -2 & 4 & -3 & 1 & 0 & 0 \\ 0 & -2 & -18 & 27 & 0 & 0 & 1 & 0\\ 0 & -1 & -8 & 12 & -2 & 0 & 0 & 1 \end{array} \right) \overset{r_2\leftrightarrow{r_4}}{\rightarrow} $$ $$ \rightarrow\left(\begin{array}{cccc|cccc} 1 & -3/2 & 0 & -1/2 & -1/2 & 0 & 0 & 0\\ 0 & -1 & -8 & 12 & -2 & 0 & 0 & 1 \\ 0 & -2 & -18 & 27 & 0 & 0 & 1 & 0\\ 0 & 0 & -2 & 4 & -3 & 1 & 0 & 0 \end{array} \right) \begin{array} {l} \phantom{0} \\ -1\cdot{r_2} \\ \phantom{0} \\ \phantom{0} \end{array} \rightarrow $$ $$ \rightarrow\left(\begin{array}{cccc|cccc} 1 & -3/2 & 0 & -1/2 & -1/2 & 0 & 0 & 0\\ 0 & 1 & 8 & -12 & 2 & 0 & 0 & -1 \\ 0 & -2 & -18 & 27 & 0 & 0 & 1 & 0\\ 0 & 0 & -2 & 4 & -3 & 1 & 0 & 0 \end{array} \right) \begin{array} {l} r_1+3/2\cdot{r_2} \\ \phantom{0} \\ r_3+2r_2 \\ \phantom{0} \end{array} \rightarrow $$ $$ \rightarrow\left(\begin{array}{cccc|cccc} 1 & 0 & 12 & -37/2 & 5/2 & 0 & 0 & -3/2\\ 0 & 1 & 8 & -12 & 2 & 0 & 0 & -1 \\ 0 & 0 & -2 & 3 & 4 & 0 & 1 & -2\\ 0 & 0 & -2 & 4 & -3 & 1 & 0 & 0 \end{array} \right) \begin{array} {l} \phantom{0} \\ \phantom{0} \\ -1/2\cdot{r_3} \\ \phantom{0} \end{array} \rightarrow $$ $$ \rightarrow\left(\begin{array}{cccc|cccc} 1 & 0 & 12 & -37/2 & 5/2 & 0 & 0 & -3/2\\ 0 & 1 & 8 & -12 & 2 & 0 & 0 & -1 \\ 0 & 0 & 1 & -3/2 & -2 & 0 & -1/2 & 1\\ 0 & 0 & -2 & 4 & -3 & 1 & 0 & 0 \end{array} \right) \begin{array} {l} r_1-12r_3 \\ r_2-8r_3 \\ \phantom{0} \\ r_4+2r_3 \end{array} \rightarrow $$ $$ \rightarrow\left(\begin{array}{cccc|cccc} 1 & 0 & 0 & -1/2 & 53/2 & 0 & 6 & -27/2\\ 0 & 1 & 0 & 0 & 18 & 0 & 4 & -9 \\ 0 & 0 & 1 & -3/2 & -2 & 0 & -1/2 & 1\\ 0 & 0 & 0 & 1 & -7 & 1 & -1 & 2 \end{array} \right) \begin{array} {l} r_1+1/2\cdot{r_4} \\ \phantom{0} \\ r_3+3/2\cdot{r_4} \\ \phantom{0} \end{array} \rightarrow $$ $$ \rightarrow\left(\begin{array}{cccc|cccc} 1 & 0 & 0 & 0 & 23 & 1/2 & 11/2 & -25/2\\ 0 & 1 & 0 & 0 & 18 & 0 & 4 & -9\\ 0 & 0 & 1 & 0 & -25/2 & 3/2 & -2 & 4\\ 0 & 0 & 0 & 1 & -7 & 1 & -1 & 2 \end{array}\right) $$

Из последней матрицы получаем ответ:

$$ A^{-1} =\left(\begin{array}{cccc} 23 & 1/2 & 11/2 & -25/2\\ 18 & 0 & 4 & -9\\ -25/2 & 3/2 & -2 & 4\\ -7 & 1 & -1 & 2 \end{array}\right) $$

Ответ: $ A^{-1} =\left(\begin{array}{cccc} 23 & 1/2 & 11/2 & -25/2\\ 18 & 0 & 4 & -9\\ -25/2 & 3/2 & -2 & 4\\ -7 & 1 & -1 & 2 \end{array}\right) $.{-1}$, если $A=\left(\begin{array}{ccc} 1 & -2 & 5\\ -2 & 5 & -13\\ -3 & 4 & -9\end{array} \right)$.

Решение

В данном примере применим метод Гаусса.

$$ \left(\begin{array}{ccc|ccc} 1 & -2 & 5 & 1 & 0 & 0\\ -2 & 5 & -13 & 0 & 1 & 0\\ -3 & 4 & -9 & 0 & 0 & 1\end{array} \right) \begin{array} {l} \phantom{0} \\ r_2+2r_1 \\ r_3+3r_1 \end{array} \rightarrow\\ $$ $$ \rightarrow\left(\begin{array}{ccc|ccc} 1 & -2 & 5 & 1 & 0 & 0\\ 0 & 1 & -3 & 2 & 1 & 0\\ 0 & -2 & 6 & 3 & 0 & 1\end{array} \right) \begin{array} {l} \phantom{0} \\ \phantom{0} \\ r_3+2r_2 \end{array} \rightarrow \left(\begin{array}{ccc|ccc} 1 & -2 & 5 & 1 & 0 & 0\\ 0 & 1 & -3 & 2 & 1 & 0\\ 0 & 0 & 0 & 7 & 2 & 1\end{array}\right) $$

В матрице до черты появилась нулевая строка.{-1}$ не существует.

Ответ: обратной матрицы не существует.

Метод обратной матрицы и как найти обратную матрицу

Решать систему линейных уравнений с помощью обратной матрицы («найти обратную матрицу») — это не самый удобный способ, но он существует. Нахождение обратной матрицы применимо, если определитель, будучи составлен из коэффициентов при переменных, 0.

Для примера возьмем опять же знакомую нам систему:

Запишем эту систему в матричной форме

A * X = B

Будем искать матицу A1, обратную к матрице А, с помощью
метода Гаусса

Для этого запишем расширенную матрицу, в которой слева будет находиться наша исходная
матрица А, а справа — единичная.

Используя метод Гаусса, постепенно приведем нашу исходную к единичной матрице. Это
преобразование применим ко всей расширенной матрице.

После приведения левой части расширенной матрицы к единичной, справа окажется матрица,
обратная к нашей исходной, а проследить последовательность приведения левой части расширенной
матрицы к единичной нам помогут серые выделенные элементы.

Рассмотрим столбец 1.

Преобразования матрицы удобнее производить в целых числах, для этого следует прибавить
соответствующие элементы строки 1 к элементам строки 2:

Прибавим соответствующие элементы строки 1 к элементам строки 2, помножив их на -3.

Это приведет к тому, что в левой части расширенной матрицы, все элементы расположенные ниже
главной диагонали = 0.

Проведем аналогичные преобразования, но уже применительно к элементам матрицы,
расположенными выше главной диагонали.

Рассмотрим столбец 2.

Разделим элементы строки 2 на -30.

Прибавим к элементам строки 1 соответствующие элементы строки 2, помноженные на -13.

Элементы строки 1 разделим на -1.

Запишем обратную матрицу.

Теперь вернемся к уравнению, записанному нами в матричной форме.

A * X = B

Умножим обе части уравнения на A1

Произведение исходной матрицы на обратную дает единичную матрицу,
т.е. A1 * A = Е, следовательно

X = A1 * B

Ответ :


Если материал был полезен, вы можете отправить донат или поделиться данным материалом в социальных сетях:

Раздел 2. Методы решения систем алгебраических уравнений

Тема 2.I методы решения систем линейных уравнений

Методы решения систем линейных алгебраических уравнений разделяются на точные и итерационные. Точные методы представляют собой конечные алгоритмы для вычисления корней системы. Это — метод обратной матрицы, метод Крамера, метод Гаусса, метод Жордана-Гаусса, метод главных элементов, метод квадратных корней и др. Несмотря на то, что методы называются точными, результаты вычислений имеют погрешности вследствие неизбежных округлений при выполнении действий.

Итерационные методы позволяют получать корни системы с заданной точностью путем сходящихся бесконечных процессов. К их числу относятся: метод простой итерации, метод Зейделя, метод релаксации и др. Эффективность применения итерационных методов существенно зависят от удачного выбора начального приближения и быстроты сходимости процесса.

Все вышеперечисленные методы (точные и приближенные) имеют свои преимущества и недостатки и поэтому для выбора оптимального (лучшего) метода для решения конкретной системы линейных уравнений требуются знания пользователя.

А. Пусть дана система nлинейных уравнений сnнеизвестными

Запишем эту систему в матричном виде

х =

или в общем матричном виде

А∙Х=В,

где: А— матрица коэффициентов;

Х– вектор искомых параметров;

В– вектор свободных членов.

Рассмотрим решение линейной системы уравнений различными методами.

Метод обратной матрицы

Дана система линейных уравнений

А∙Х=В.

Умножим правую и левую части системы на обратную матрицу А-1

А-1∙А∙Х= А-1∙В

Так как А-1∙А=Е,тоЕ∙Х= А-1∙В ЕХ=Х

Так как ЕХ=Хто,Х= А-1∙В.

Таким образом данный метод заключается в нахождении обратной матрицы коэффициентов А-1и ее умножении на вектор свободных членов В. Нахождение обратной матрицыА-1при порядкеn>4 требует много времени, поэтому метод обратной матрицы редко употребляется.

Метод Крамера

Известно, что

.

Отсюда

,

где , , …,

где

Итак, метод Крамера заключается в вычислении (n+1)-го определителя (∆1, ∆2, ∆3, …, ∆n) для матрицn-го порядка. Если число велико, то вычисление определителей является трудоемкой задачей.

Наиболее распространенным способом решения системы линейных уравнений является метод последовательного исключения неизвестных или метод Гаусса.

Метод Гаусса

Рассмотрим на простейшем примере известный со школы способ исключения неизвестных при решении систем уравнений. Пусть дана система:

Умножим первое уравнение на такой коэффициент , чтобы в обоих уравнениях коэффициент прих1стал бы одинаковым

Теперь вычтем его из второго уравнения, т.е.

-2х1+х2=7

Мы выполнили операцию исключения неизвестной х1 из второго уравнения. Запишем систему уравнения после этого исключения в следующем виде. Первое уравнение записываем в исходном виде.

Второе уравнение содержит лишь одно неизвестное, которое легко вычисляется х2=3. Подставив полученное значениех2в первое уравнение, можем вычислить и первое неизвестноех1.

Проведенные действия и составляют сущность метода Гаусса. Рассмотрим преобразования по методу Гаусса для системы уравнений n-го порядка.

а11х1+ а12х2+ … +а1nхn=b1

х

а21х1+ а22х2+ … +а2nхn=b2

а31х1+ а32х2+ … +а3nхn=b3

… … … …

аn1х1+ аn2х2+ … +аnnхn=bn

Вычтем из второго уравнения первое, умноженное на .

При этом во втором уравнении будет уничтожен коэффициент при х1.

Затем из третьего уравнения также вычтем первое, умноженное на .

Проделав аналогичные преобразования с остальными уравнениями системы, превратим в нуль все коэффициенты первого столбца, кроме элемента а11. Получим следующую систему:

а11х1+ а12х2+ а13х3+… +а1nхn=b1

… … … …

Затем при помощи второго уравнения преображенной системы исключим из третьего, четвертого и т.д. уравнений коэффициенты второго столбца лежащие ниже

а11х1+ а12х2+ а13х3+… +а1nхn=b1

… … …

Последовательно продолжая этот процесс, исключим из системы все коэффициенты, лежащие ниже главной диагонали. В результате получим треугольную систему уравнений.

а11х1+ а12х2+а13х3+… +а1nхn=b1

… … …

Процесс получения треугольной системы называется “прямым ходом” по методу Гаусса. Треугольная система легко решается “обратным ходом”. Из последнего уравнения определяется последнее неизвестное . Затем из предпоследнего уравнения постановкой найденного значенияхnопределяетсяхn-1. После решения системы уравнений методом Гаусса необходимо делать проверку, подставляя в исходные уравнения найденные значения переменных хi(i= 1, …,n).

При решении системы линейных уравнений методом Гаусса все вычисления можно поместить в следующую таблицу. Рассмотрим таблицу на примере решения системы уравнений третьего порядка.

№ шага преобразований

х х1

х х2

х х3

1)

а11

а12

а13

b1

:а11

0

а21

а22

а23

b2

а31

а32

а33

b3

1

1 2)

:

1

2 3)

:

1

Уравнения 1), 2) и 3) составляют искомую треугольную матрицу после “прямого хода”. Число шагов преобразований в “прямом ходе” методом Гаусса равно n-1.

Коэффициенты а11,,- называются “ведущими” элементами.

При “обратном ходе” можно использовать строки таблицы, содержащие единицы, т.е. вспомогательные уравнения. Имеем далее

ПРИМЕР:

№ шага

х1

х2

х3

B

4

1

2

12

: 4

0

2

8

4

30

1

2

4

17

1

0,25

0,5

3

х 2

х 1

1

0

7,5

3

24

0

1,75

3,5

14

1

0,4

3,2

х 1,75

2

0

2,8

8,4

3

1

3

Треугольная система

4х1+х2+2х3=12

7,5х2+3х3=24

2,8х3=8,4

или

х1+0,25х2+0,5х3=3

х2+0,4х3=3,2

х3=3

Обратный ход

х2=3,2-0,4∙3=2

х1=3-0,25∙2-0,5∙3=1

Вычисление определителя методом Гаусса

(третий способ, без вывода)

Определитель матицы Аравен произведению всех “ведущих” элементов при преобразовании ее по методу Гаусса.

Для вычисления определителя матрицы Авыполняется только “прямой” ход методом Гаусса, причем столбец свободных членовВстановится излишним.

ПРИМЕР: дана матрица

detА=4∙7,5∙2,8=84

Вычисление обратной матрицы методом Гаусса

АА-1 =Е

Матрицы АиЕизвестны, требуется определитьА-1. Обозначим столбцы матрицыА-1черезх1,х2, …,хnт.е.

Столбцы для матрицы Е обозначим через Е1,Е2, …,Еn

Тогда можем записать nсистем уравнений

Ах1=Е1

Ах2=Е2

Ахn=Еn

Развернем первое матричное уравнение Ах1=Е1

х =

Другие матричные уравнения аналогичны.

Следовательно, для получения обратной матрицы А-1достаточно выполнитьnрешений методом Гаусса систем линейных уравнений с разными правыми частями —yстолбцами матрицыЕ.

Полученные решения х1,х2, …,хnбудут столбцами искомой обратной матрицыА-1.

Трангуляции матрицы

Квадратную матрицу Аможно представить как произведение двух треугольных матрицА=LW, где

L– нижняя треугольная матрица,

W– верхняя треугольная матрица.

Матрица Wвычисляется при прямом ходе Гаусса

а11а12а13а1n

0 …

0 0 …

… … … … …

0 0 0 …

У матрицы Lнаоборот все элементы выше главной диагонали нулевые. Остальные элементы матрицыLвычисляются в результате деления элементов по столбцам, полученных при том же прямом ходе Гаусса, на ведущие элементы. Сначала вычисляются элементы первого столбца матрицыLделением на ведущий элемента11, затем после первого шага “прямым ходом” метода Гаусса вычисляются элементы второго столбца, начиная с диагонального, делением на ведущий элемента11и т.д.

Требуется решить системы уравнений

Ах=В

Так как А=LWтоLWх=В

Обозначим =Z

Тогда вместо системы Ах=Вможем записать ей эквивалентную

LZ=В

Wx=Z (5)

Решение эквивалентной системы с треугольными матрицами LиWзанимает гораздо меньше времени, чем решение исходной системыАх=В. Это обстоятельство очень важно при необходимости решать систему уравнений многократно при одной и той же матрицеАи разных векторах свободных членовВ, что обычно имеет место при расчетах режимов работы электрических систем. Триангуляция же матрицыАпроводится только один раз.

То есть элементы матрицыА– это, как правило, параметры схемы замещения эл. системы,В– вектор узловых токов или мощностей. Часто ставится задача определения параметров большего числа режимов при изменении токов или мощностей потребителей в узлах при неизменной схеме замещения. Если триангуляция матрицыАосуществлена, то можно быстро пользуясь системой (5) посчитать необходимые режимы, меняя в этой системе вектор В. Для каждого режима сначала решается треугольная подсистемаLZ=ВотносительноZпоследовательной подстановкой в уравнения подсистемы найденных значений неизвестных из предыдущих уравнений, начиная сZ1

Z1 =b1

=b2

=b3

… … … … …

=bn

Значение Z1уже известно из первого уравнения,Z2определяется из второго уравнения подстановкой в него значенияZ1и т.д. Определяются всеZ. Затем аналогично решается вторая треугольная подсистемаWx=Zпутем обратной подстановки, начиная схn(аналогично обратному ходу методом Гаусса).

Метод Жордана-Гаусса

Метод Жордана-Гаусса называют еще методом Гаусса без обратного хода. Сущность его состоит в том, что на втором шаге переменная исключается из всех уравнений, кроме второго, на третьем шаге исключается также из всех уравнений, кроме третьего и т.д. После шагов в каждом уравнении остается одна неизвестная, т.е. получим решение системы таким образом, исключение переменных по методу Жордана-Гаусса эквивалентно преобразованию матрицы коэффициентов в единичную. Рассмотрим таблицу вычислений по методу Жордана-Гауса.

№ шага преобразований

A

C

B

0

а11

a12

a1n

b1

а21

a22

a2n

b2

а31

a32

a3n

b3

аn1

an2

ann

bn

1

1

0

0

0

n

1

0

0

0

1

0

..

..

0

0

1

Нахождение обратной матрицы методом Жордана-Гаусса

Рассмотрим вычислительную процедуру определителя А-1на конкретном примере.

А∙А-1=Е

Пусть

Ах1х2 х3Е1Е2Е3

х =

№ шага

преобразования

х х1

х х2

х х3

Е1

Е2

Е3

4

1

2

1

0

0

0

2

8

4

0

1

0

1

2

4

0

0

1

1

0,25

0,5

0,25

0

0

1

0

7,5

3

-0,5

1

0

0

1,75

3,5

-0,25

0

1

1

0

0,4

0,2667

-0,0333

0

2

0

1

0,4

-0,0667

0,1333

0

0

0

2,8

-0,1333

-0,2333

1

1

0

0

0,2857

0

-0,1428

3

0

1

0

-0,0477

0,1666

-0,1428

0

0

1

-0,0476

-0,0833

0,357

Проверка:

х =

По такой же вычислительной схеме можно вычислять значения переменных х1,х2,х3, …,хпри одной матрице коэффициентаАи разных столбцах свободных членовВ1,В2,В3, …,Вn.

Недостатки метода Гаусса (его недостатки и способы их устранения)

  1. Если определитель матрицы Амал, то из-за ошибок округлений сильно снижается точность получения искомых корней.

  2. Метод Гаусса требует, чтобы диагональные элементы в процессе исключения переменных не были равны нулю (т.к. строки делятся на них). Поэтому часть применяют метод Гаусса с выбором главного элемента, который заключается в следующем. При обращении в нуль элементов первого столбца из всей матрицы выбирается наибольший элемент и затем в нуль элементы второго столбца, рассматривается сокращенная матрица (путем вычеркивания в уже полученной системе первого уравнения) и в ней наибольший элемент переставляется на ее первое место и т.д.

  3. Метод Гаусса требует большего объема памяти ЭВМ по сравнению с итерационными методами. Существуют различные приемы по сокращению занимаемой памяти ЭВМ при решении методом Гаусса электроэнергетических задач.

Например, необходимо решить систему

Yy Uy=Iy

при использовании метода узловых напряжений.

Перенумерация

  1. Экономичность памяти

  2. Сокращение времени счета

Y=

1

2

3

4

5

6

7

8

9

10

1

2

3

4

5

6

7

8

9

10

1

+

+

+

1

+

+

+

3

2

+

+

+

2

+

+

+

6

3

+

+

3

+

+

+

10

4

+

+

+

4

+

+

+

14

5

+

+

5

+

+

17

6

+

+

+

6

+

+

+

+

+

21

7

+

+

+

7

+

+

+

+

24

8

+

+

+

+

8

+

+

26

9

+

+

+

+

+

9

+

+

+

27

10

+

+

10

+

+

+

(30 х 70)

Я=10 х 10=100 Я=52 (26)

В ряде случаев для нахождения корней системы линейных уравнений удобнее пользоваться приближенными итерационными методами (или методами последовательных приближений).

Нахождение обратной матрицы: три алгоритма и примеры

Нахождение обратной матрицы — процесс, который состоит из достаточно простых действий. Но эти действия повторяются так часто, что процесс получается довольно продолжительным. Главное — не потерять внимание при решении.

При решении наиболее распространённым методом — алгебраических дополнений — потребуется:

При решении примеров мы разберём эти действия подробнее. А пока узнаем, что гласит теория об обратной матрице.

Для обратной матрицы существует уместная аналогия с обратным числом. Для каждого числа a, не равного нулю, существует такое число b, что произведение a и b равно единице: ab = 1. Число b называется обратным для числа b. Например, для числа 7 обратным является число 1/7, так как 7*1/7=1.

Обратной матрицей, которую требуется отыскать для данной квадратной матрицы А, называется такая матрица

,

произведение на которую матрицы А справа является единичной матрицей, т.е,
.                (1)

Единичной матрицей называется диагональная матрица, у которой все диагональные элементы равны единице.

Нахождение обратной матрицы — задача, которая чаще решается двумя методами:

  • методом алгебраических дополнений, при котором, как было замечено в начале урока, требуется находить определители, миноры и алгебраические дополнения и транспонировать матрицы;
  • методом исключения неизвестных Гаусса, при котором требуется производить элементарные преобразования матриц (складывать строки, умножать строки на одно и то же число и т. д.).

Для особо любознательных существуют и другие методы, например, метод линейных преобразований. На этом уроке разберём три упомянутых метода и алгоритмы нахождения обратной матрицы этими методами.

Теорема. Для каждой неособенной (невырожденной, несингулярной) квадратной матрицы можно найти обратную матрицу, и притом только одну. Для особенной (вырожденной, сингулярной) квадратной матрицы обратная матрица не существует.

Квадратная матрица называется неособенной (или невырожденной, несингулярной), если её определитель не равен нулю, и особенной (или вырожденной, сингулярной), если её определитель равен нулю.

Обратная матрица может быть найдена только для квадратной матрицы. Естественно, обратная матрица также будет квадратной и того же порядка, что и данная матрица. Матрица, для которой может быть найдена обратная матрица, называется обратимой матрицей.

На сайте есть онлайн калькулятор для нахождения обратной матрицы. Вы можете открыть его в новом окне уже сейчас, если держите перед собой ваши собственные задания. А мы разберём несколько разминочных.

Для неособенной квадратной матрицы А обратной является матрица

,  (2)

где — определитель матрицы А, а — матрица, союзная с матрицей А.

Разберём ключевые понятия, которые потребуются для решения задач — союзная матрица, алгебраические дополнения и транспонированная матрица.

Пусть существует квадратная матрица A:

Транспонированная относительно матрицы A матрица A’ получается, если из строк матрицы A сделать столбцы, а из её столбцов — наоборот, строки, то есть заменить строки столбцами:

Остановимся на минорах и алгебраических дополнениях.

Пусть есть квадратная матрица третьего порядка:

.

Её определитель:

Вычислим алгебраическое дополнение элемента , то есть элемента 2, стоящего на пересечении первой строки и второго столбца.

Для этого нужно сначала найти минор этого элемента. Он получается вычёркиванием из определителя строки и столбца, на пересечении которых стоит указанный элемент. В результате останется следующий определитель, который и является минором элемента :

.

Алгебраическое дополнение элемента получим, если умножим , где i — номер строки исходного элемента, а k — номер столбца исходного элемента, на полученный в предыдущем действии минор этого исходного элемента. Получаем алгебраическое дополнение элемента :

.

По этой инструкции нужно вычислить алгебраические дополнения всех элементов матрицы A’, транспонированной относительно матрицы матрица A.

И последнее из значимых для нахождение обратной матрицы понятий. Союзной с квадратной матрицей A называется матрица того же порядка, элементами которой являются алгебраические дополнения соответствующих элементов определителя матрицы , транспонированной относительно матрицы A. Таким образом, союзная матрица состоит из следующих элементов:

Алгоритм нахождения обратной матрицы методом алгебраических дополнений

1. Найти определитель данной матрицы A. Если определитель равен нулю, нахождение обратной матрицы прекращается, так как матрица вырожденная и обратная для неё не существует.

2. Найти матрицу, транспонированную относительно A.

3. Вычислить элементы союзной матрицы как алгебраические дополнения марицы, найденной на шаге 2.

4. Применить формулу (2): умножить число, обратное определителю матрицы A, на союзную матрицу, найденную на шаге 4.

5. Проверить полученный на шаге 4 результат, умножив данную матрицу A на обратную матрицу. Если произведение этих матриц равно единичной матрицы, значит обратная матрица была найдена верно. В противном случае начать процесс решения снова.


Пример 1. Для матрицы

найти обратную матрицу.

Решение. Для нахождения обратной матрицы необходимо найти определитель матрицы А . Находим по правилу треугольников:

Следовательно, матрица А – неособенная (невырожденная, несингулярная) и для неё существует обратная.

Найдём матрицу, союзную с данной матрицей А.

Найдём матрицу , транспонированную относительно матрицы A:

Вычисляем элементы союзной матрицы как алгебраические дополнения матрицы, транспонированной относительно матрицы A:

Следовательно, матрица , союзная с матрицей A, имеет вид

Замечание. Порядок вычисления элементов и транспонирования матрицы может быть иным. Можно сначала вычислить алгебраические дополнения матрицы A, а затем транспонировать матрицу алгебраических дополнений. В результате должны получиться те же элементы союзной матрицы.

Применяя формулу (2), находим матрицу, обратную матрице А:

Проверить решение можно с помощью онлайн калькулятора для нахождения обратной матрицы.

Алгоритм нахождения обратной матрицы методом исключения неизвестных Гаусса

1. К матрице A приписать единичную матрицу того же порядка.

2. Полученную сдвоенную матрицу преобразовать так, чтобы в левой её части получилась единичная матрица, тогда в правой части на месте единичной матрицы автоматически получится обратная матрица. Матрица A в левой части преобразуется в единичную матрицу путём элементарных преобразований матрицы.

2. Если в процессе преобразования матрицы A в единичную матрицу в какой-либо строке или в каком-либо столбце окажутся только нули, то определитель матрицы равен нулю, и, следовательно, матрица A будет вырожденной, и она не имеет обратной матрицы. В этом случае дальнейшее нахождение обратной матрицы прекращается.

Пример 2. Для матрицы

найти обратную матрицу.

Решение. Составляем сдвоенную матрицу

и будем её преобразовывать, так чтобы в левой части получилась единичная матрица. Начинаем преобразования.

Умножим первую строку левой и правой матрицы на (-3) и сложим её со второй строкой, а затем умножим первую строку на (-4) и сложим её с третьей строкой, тогда получим

.

Чтобы по возможности не было дробных чисел при последующих преобразованиях, создадим предварительно единицу во второй строке в левой части сдвоенной матрицы. Для этого умножим вторую строку на 2 и вычтем из неё третью строку, тогда получим

.

Сложим первую строку со второй, а затем умножим вторую строку на (-9) и сложим её с третьей строкой. Тогда получим

.

Разделим третью строку на 8, тогда

.

Умножим третью строку на 2 и сложим её со второй строкой. Получается:

.

Переставим местами вторую и третью строку, тогда окончательно получим:

.

Видим, что в левой части получилась единичная матрица, следовательно, в правой части получилась обратная матрица . Таким образом:

.

Можно проверить правильность вычислений, умножим исходную матрицу на найденную обратную матрицу:

.

В результате должна получиться обратная матрица.

Проверить решение можно с помощью онлайн калькулятора для нахождения обратной матрицы.

Пример 3. Для матрицы

найти обратную матрицу.

Решение. Составляем сдвоенную матрицу

и будем её преобразовывать.

Первую строку умножаем на 3, а вторую на 2, и вычитаем из второй, а затем первую строку умножаем на 5, а третью на 2 и вычитаем из третьей строки, тогда получим

.

Первую строку умножаем на 2 и складываем её со второй, а затем из третьей строки вычитаем вторую, тогда получим

.

Видим, что в третьей строке в левой части все элементы получились равными нулю. Следовательно, матрица вырожденная и обратной матрицы не имеет. Дальнейшее нахождение обратной марицы прекращаем.

Проверить решение можно с помощью онлайн калькулятора для нахождения обратной матрицы.

Матрицы теснейшим образом связаны с системами линейных уравнений. Каждой матрице соответствует система линейных уравнений, коэффициенты в которой есть элементы матрицы. И наоборот, системе линейных уравнений соответствует некоторая матрица.

Поэтому существует метод линейных преобразований для нахождения обратной матрицы. Для решения задач нам будет достаточно знать, что линейное преобразование — это система линейных уравнений, вид которой будет приведён ниже в алгоритме.

Алгоритм нахождения обратной матрицы методом линейных преобразований

1. Для данной невырожденной матрицы A составить линейное преобразование — систему линейных уравнений вида

,

где aij — элементы матрицы A.

2. Решить полученную систему относительно y — найти для предыдущего линейного преобразование обратное линейное преобразование

,

в котором Aij — алгебраические дополнения элементов матрицы A, Δ — определитель матрицы A. Внимание! Алгебраические дополнения располагаются как в транспонированной матрице, то есть для элементов строки — в столбце, а для элементов столбца — в строке.

3. Находим коэффициенты при y: , которые и будут элементами матрицы, обратной для матрицы A.

4. Пользуясь элементами, найденными на шаге 3, записать найденную обратную матрицу.

Наиболее наблюдательные могли заметить, что по сути метод линейных преобразований — это тот же метод алгебраических преобразований (союзной матрицы), но с другой формой записи. Для кого-то метод линейных преобразований может оказаться более удобным как более компактный.

Пример 4. Найти обратную матрицу для матрицы

.

Сначала проверим, не равен ли нулю определитель данной матрицы. Он не равен нулю, следовательно, обратная матрица существует.

Для данной матрицы записываем линейное преобразование:

.

Находим линейное преобразование, обратное предыдущему, для этого потребуется находить алгебраические дополнения (урок откроется в новом окне). Запишем обратное линейное преобразование:

Коэффициенты при иксах в обратном линейном преобразовании — это элементы обратной матрицы для матрицы A. Таким образом нашли обратную матрицу:

Проверить решение можно с помощью онлайн калькулятора для нахождения обратной матрицы.

Начало темы «Матрицы»

Другие темы линейной алгебры

Обратная матрица методом Гаусса | Численные методы на Python

Для получения матрицы будем исходить из того, что она является решением уравнения , где — единичная матрица. Представим искомую матрицу как набор векторов-столбцов img а единичную матрицу как набор единичных векторов img Матричное уравнение в соответствии с правилами умножения матриц возможно заменить не связанной между собой системой уравнений img Каждое из этих уравнений может быть решено методом Гаусса. Заметим то обстоятельство, что все СЛАУ имеют одну и ту же матрицу коэффициентов, поэтому

Суть метода Гаусса-Жордана заключается в том, что если с единичной матрицей провести элементарные преобразования, которыми невырожденная квадратная матрица приводится к , то получится обратная матрица . Для отображения всех элементарных преобразований, совершающихся над матрицей , на единичную матрицу , удобно «склеить» две матрицы в одну. Если матрицы и имели размер , то склеенная матрица будет иметь размер . Таким образом, все преобразования над матрицей (перестановка строк, умножение строки на константу) будут автоматически совершаться и над единичной матрицей). Обозначим склеенную матрицу как Приведение матрицы к единичной совершается в два этапа: на первом этапе будем двигаться «сверху-вниз», получая в итоге треугольную матрицу, с единицами на главной диагонали и нулями ниже. На втором этапе будем двигаться «снизу-вверх», преобразуя элементы, лежащие выше главной диагонали к нулю, тем самым добиваясь единичной матрицы в результате.

Прямой ход

Пусть матрица имеет размер . Тогда на первом этапе будет совершено шагов. На каждом шаге необходимо совершить три действия.

  1. Поменять местами строки и , в случае если . Этот шаг необходим для исключения ситуации нахождения на главной диагонали 0.
  2. Преобразовать элемент, стоящий на главной диагонали к 1. Для этого необохдимо домножить всю строку на .
  3. Обнулить все нижележащие элементы столбца. Для этого вычтем из каждой нижележащей строки c индексом результат умножения элемента c текущей строкой. Напомним, что в результате шага 2 первый элемент 1, поэтому первым элементом вычитаемой строки будет являться значение первого элемента строки , что и даст 0 в элементе

Обратный ход

Теперь необходимо преобразовать матрицу так, чтобы все элементы n-ого столбца выше стали равны нулю. Для этого прибавляем к n-1 строке соответсвующие элементы n-ой строки, умноженные на . К n-2 строке прибавляем соответсвующие элементы (n-1)-ой строки, умноженные на и т.д. Аналогичные действия необходимо совершить над оставшимися строками.

Реализация

Приведем реализацию функции inverse, принимающей в качестве аргумента исходную матрицу коэффициентов, и возвращающую матрицу, обратную к исходной. Склеим матрицу коэффициентов с единичной матрицей с помощью функции hstack

m = np.hstack((matrix_origin, 
                np.matrix(np.diag([1.0 for i in range(matrix_origin.shape[0])]))))

Прямой ход


    for k in range(n):
        
        swap_row = pick_nonzero_row(m, k)
        if swap_row != k:
            m[k, :], m[swap_row, :] = m[swap_row, :], np.copy(m[k, :])
        
        if m[k, k] != 1:
            m[k, :] *= 1 / m[k, k]
        
        for row in range(k + 1, n):
            m[row, :] -= m[k, :] * m[row, k]

Для проверки первгого условия реализуем вспомогательную функцию pick_non_zero_row, возвращающую индекс первой строки, в которой элемент интересующего нас столбца не равен нулю

def pick_nonzero_row(m, k):
    while k < m.shape[0] and not m[k, k]:
        k += 1
    return k

Обратный ход


    for k in range(n - 1, 0, -1):
        for row in range(k - 1, -1, -1):
            if m[row, k]:
                
                m[row, :] -= m[k, :] * m[row, k]

Возвратим преобразованную единичную матрицу, т.е. вторую часть «склееного» массива, воспользовавшись функцией hsplit

return np.hsplit(m, n // 2)[1]

Метод Гаусса. Как найти обратную матрицу с помощью элементарных преобразований?

Однажды некто Жордано (не путать с Джордано Бруно) сел решать очередную систему уравнений. Он любил этим заниматься и в свободное время совершенствовал свои навыки. Но вот настал момент, когда ему наскучили все методы решения и метод Гаусса в том числе. Предположим, дана система с тремя уравнениями, тремя неизвестными и записана её расширенная матрица . В наиболее распространенном случае получаются стандартные ступеньки , и так каждый день…. Одно и то же – как беспросветный ноябрьский дождь.

На некоторое время развевает тоску другой способ приведения матрицы к ступенчатому виду: , причём он совершенно равноценен и может быть не удобен только по причине субъективного восприятия. Но всё рано или поздно приедается…. И подумал тогда математик – а зачем вообще мучиться с обратным ходом гауссовского алгоритма? Не проще ли сразу получить ответ  с помощью дополнительных элементарных преобразований?

Для освоения данного урока «чайникам» придётся пойти путём Жордано и прокачать элементарные преобразования хотя бы среднего уровня, прорешав, минимум, 15-20 соответствующих заданий. Поэтому если вы смутно понимаете, о чём идёт разговор и/или у вас возникнет недопонимание чего-либо по ходу занятия, то рекомендую ознакомиться с темой в следующем порядке:

Метод Гаусса для чайников;

Несовместные системы и системы с общим решением;

Ранг матрицы;

Однородные системы.

Ну, и совсем замечательно, если отработаны элементарные преобразования определителя.

Как все поняли, метод Жордано-Гаусса представляет собой модификацию метода Гаусса и с реализацией основной, уже озвученной выше идеи, мы встретимся на ближайших экранах. Кроме того, в число немногочисленных примеров данной статьи вошло важнейшее приложение – нахождение обратной матрицы с помощью элементарных преобразований.

Не мудрствуя лукаво:

Пример 1

Решить систему методом Жордано-Гаусса

Решение: это первое задание урока Метод Гаусса для чайников, где мы 5 раз трансформировали расширенную матрицу системы и привели её к ступенчатому виду:

Теперь вместо обратного хода в игру вступают дополнительные элементарные преобразования. Сначала нам необходимо получить нули на этих местах: , 
а потом ещё один ноль здесь: .

Идеальный с точки зрения простоты случай:

(6) Ко второй строке прибавили третью строку. К первой строке прибавили третью строку.

(7) К первой строке прибавили вторую строку, умноженную на –2.

Не могу удержаться от иллюстрации итоговой системы:

Ответ

Предостерегаю читателей от шапкозакидательского настроения – это был простейший демонстрационный пример. Для метода Жордано-Гаусса характерны свои специфические приёмы и не самые удобные вычисления, поэтому, пожалуйста, настройтесь на серьёзную работу:

Пример 2

Решить систему линейных уравнений методом Жордано-Гаусса.

Решение: первая часть задания хорошо знакома:

(1) Ко второй строке прибавили первую строку, умноженную на –1. К третьей строке прибавили первую строку, умноженную на 3. К четвертой строке прибавили первую строку, умноженную на –5.

(2) Вторую строку разделили на 2, третью строку разделили на 11, четвёртую строку разделили на 3.

(3) Вторая и третья строки пропорциональны, 3-ю строку удалили. К четвёртой строке прибавили вторую строку, умноженную на –7

(4) Третью строку разделили на 2.

Очевидно, что система имеет бесконечно много решений, и наша задача – привести её расширенную матрицу к виду .

Как действовать дальше? Прежде всего, следует отметить, что мы лишились вкусного элементарного преобразования – перестановки строк. Точнее говоря, переставлять-то их можно, но в этом нет смысла. И далее целесообразно придерживаться следующего шаблона:

Находим наименьшее общее кратное чисел третьего столбца (1, –1 и 3), т.е. – наименьшее число, которое бы делилось без остатка и на 1, и на –1 и на 3. В данном случае, это, конечно же, «тройка». Теперь в третьем столбце нам нужно получить одинаковые по модулючисла, и этими соображениями обусловлено 5-ое преобразование матрицы:

(5) Первую строку умножаем на –3, вторую строку умножаем на 3. Вообще говоря, первую строку можно было умножить тоже на 3, но это было бы менее удобно для следующего действия. К хорошему привыкаешь быстро:


(6) Ко второй строке прибавили третью  строку. К первой строке прибавили третью строку.

(7) Во втором столбце два ненулевых значения (24 и 6) и нам снова нужно получитьодинаковые по модулю числа. В данном случае всё сложилось довольно удачно – наименьшее кратное 24, и эффективнее всего умножить вторую строку на –4.

(8) К первой строке прибавили вторую.

(9) Заключительный штрих: первую строку разделили на –3, вторую строку разделили на –24 и третью строку разделили на 3. Это действие выполняется В ПОСЛЕДНЮЮ ОЧЕРЕДЬ! Никаких преждевременных дробей!

В результате элементарных преобразований получена эквивалентная исходной система:

Элементарно выражаем базисные переменные через свободную:

и записываем:

Ответ: общее решение: 

В подобных примерах применение рассмотренного алгоритма чаще всего оправдано, поскольку обратный ход метода Гаусса обычно требует трудоёмких и неприятных вычислений с дробями.

И, разумеется, крайне желательна проверка, которая выполняется по обычной схеме, рассмотренной на уроке Несовместные системы и системы с общим решением.

Для самостоятельного решения:

Пример 3

Найти базисное решение с помощью элементарных преобразований

Такая формулировка задачи предполагает использование метода Жордано-Гаусса, и в образце решения матрица приводится к стандартному виду  с базисными переменными . Однако всегда держите на заметке, что в качестве базисных можно выбрать и другие переменные. Так, например, если в первом столбце громоздкие числа, то вполне допустимо привести матрицу к виду   (базисные переменные ), или к виду  (базисные переменные ), или даже к виду  с базисными переменными . Существуют и другие варианты.

Но всё-таки это крайние случаи – не стОит лишний раз шокировать преподавателей своими знаниями, техникой решения и уж тем более не надо выдавать экзотических жордановсих результатов вроде . Впрочем, бывает трудно удержаться от нетипового базиса, когда в исходной матрице, скажем, в 4-ом столбце есть два готовых нуля.

Примечание: термин «базис» имеет алгебраический смысл и понятиегеометрического базиса здесь не при чём!

Если в расширенной матрице данных размеров вдруг обнаруживается пара линейно зависимых строк, то её следует попытаться привести к привычному виду   с базисными переменными . Образец такого решения есть в Примере №7 статьи ободнородных системах линейных уравнений, причём там выбран другой базис.

Продолжаем совершенствовать свои навыки на следующей прикладной задаче:


Как найти обратную матрицу методом Гаусса?

Обычно условие формулируют сокращённо, но, по существу, здесь также работает алгоритм Жордано-Гаусса. Более простой метод нахождения обратной матрицы  для квадратной матрицы  мы давным-давно рассмотрели на соответствующем уроке, и суровой поздней осенью тёртые студенты осваивают мастерский способ решения.

Краткое содержание предстоящих действий таково: сначала следует записать квадратную матрицу  в тандеме с единичной матрицей: . Затем с помощью элементарных преобразований необходимо получить единичную матрицу слева, при этом (не вдаваясь в теоретические подробности) справа нарисуется обратная матрица. Схематически решение выглядит следующим образом:

(Понятно, что обратная матрица должна существовать)

Не хочу показаться категоричным или придирчивым, но в подавляющем большинстве источников информации, которые я видел, данная задача рассмотрена крайне плохо – нужно обладать семью пядями во лбу и потратить массу времени/нервов на тяжёлое неуклюжее решение. За годы практики мне удалось отшлифовать, не скажу, что самую лучшую, но рациональную и достаточно лёгкую методику, которая доступна всем, кто владеет арифметическими действиями!

Демо-пример: найдём обратную матрицу для матрицы  с помощью элементарных преобразований. Для этого запишем её в одной упряжке с единичной матрицей: .

А теперь об одном принципиальном моменте. По цитате известного юмориста, для русского человека есть несколько градаций запрета: «запрещено», «строго запрещено» и «категорически запрещено». Так вот, в рассматриваемой задаче КАТЕГОРИЧЕСКИ ЗАПРЕЩЕНО переставлять строки. Если в ходе решения систем мы могли выполнять данное преобразование, то здесь его полное отсутствие заметно огранивает наши возможности.

Однако не всё так плохо:

(1) Ко второй строке прибавили первую строку, умноженную на –3.

(2) К первой строке прибавили вторую строку.

(3) Вторую строку разделили на –2.

Таким образом: . Желающие могут свериться с ответом первого примера урока Как найти обратную матрицу?

Но то была очередная заманивающая задачка – в действительности решение гораздо более длительно и кропотливо. Отработаем на реальных примерах алгоритм, который я считаю наиболее выгодным. Как правило, вам будет предложена матрица «три на три»:

Пример 4

Найти обратную матрицу с помощью элементарных преобразований

Решение: присоединяем единичную матрицу и думаем над первым действием. Чтобы получить слева вверху «единицу» хочется поменять местами первую и третью строки, однако беда в том, что ПЕРЕСТАВЛЯТЬ НИЧЕГО НЕЛЬЗЯ. Поэтому используем уже знакомый по предыдущему параграфу мотив: находим наименьшее общее кратное чисел первого столбца (3, 2 и 1): 6. В этой связи:

(1) Первую строку умножаем на –2, вторую строку умножаем на 3, третью строку – на 6:

(2) Ко 2-ой и 3-ей строкам прибавили первую строку.

(3) Первую строку разделили «обратно» на –2.  Третью строку разделили на 2.

(4) Что скажешь, тут немного повезло: к третьей строке прибавили вторую строку.

(5) У второй строки сменили знак, третью строку разделили на –3.

Первая половина пути пройдена.

Далее смотрим на числа третьего столбца (2, 13, 4) и находим их наименьшее общее кратное(НОК): 52. Существует строгий алгоритм нахождения НОК, но здесь обычно хватает подбора. Ничего страшного, если взять бОльшее число, которое делится и на 2, и на 13, и на 4, например, 104. Отличие будет в более громоздких вычислениях. 

Кстати, про вычисления. Для решения данной задачи совсем не зазорно вооружиться микрокалькулятором – числа фигурируют немалые, и будет очень обидно допустить вычислительную ошибку.

Итак, на нижнем этаже получаем –52, а на двух верхних 52. Для этого:

(6) Первую строку умножаем на 26, вторую строку умножаем на 4, третью строку – на –13:

(7) К первой и второй строкам прибавили третью строку.

(8) Первую строку разделили на 13. Третью строку разделили «обратно» на –13.

(9) Наименьшее общее кратное ненулевых чисел второго столбца (8 и 44) равно 88. Первую строку умножили на 11, вторую строку умножили на –2.

(10) К первой строке прибавили вторую строку.

(11) Первую строку разделили на 3, вторую строку разделили «обратно» на –2.

(12) Теперь на главной диагонали левой матрицы целесообразно получитьнаименьшее общее кратное чисел диагонали (22, 44 и 4-х). Это число 44. Первую строку умножили на 2, третью строку умножили на 11.

(13) Каждую строку матрицы делим на 44. Данное действие выполняется в последнюю очередь!

Таким образом, обратная матрица:

Внесение и вынесение -ой, в принципе, лишние действия, но того требует протокол оформления задачи.

Ответ

Проверка выполняется по обычной схеме, рассмотренной на уроке об обратной матрице.

Продвинутые люди могут сократить и несколько видоизменить решение, но должен предупредить, отклонение от курса чревато повышенным риском допустить ошибку. По моему мнению, предложенная схема если и не самая, то одна из самых надёжных.

Аналогичное задание для самостоятельного решения:

Пример 5

Найти обратную матрицу методом Жордано-Гаусса.

Примерный образец оформления внизу страницы.

Иногда бывает удобно более короткое «модернистское» решение, которое заключается в следующем: на первом шаге всё как обычно: .

На втором шаге накатанным приёмом (через НОК чисел 2-го столбца) организуются сразу два нуля во втором столбце: . Перед данным действием особенно трудно устоять, если во 2-ом столбце нарисовались одинаковые по модулю числа, например, «единицы».

И, наконец, на третьем шаге точно так же получаем нужные нули в третьем столбце: .

Живой пример авангарда можно посмотреть во втором задании урока о решении системы в различных базисах.

Что касается размерности, то в 98-99% случаев приходится разруливать матрицу «три на три». Пару раз в пятилетку попадается лайт-версия задачи с матрицей «два на два». Алгоритм, как вы догадываетесь, аналогичный. В самом тяжелом случае через НОК чисел 1-го столбца получаем ноль слева внизу, а затем с помощью НОК чисел 2-го столбца организуем ноль вверху данного столбца. Матрица «четыре на четыре» мне встречалась всего один раз – много-много лет назад в собственной вузовской контрольной. К слову, для неё использование метода Жордано-Гаусса куда менее трудозатратно, нежели обычное решение через алгебраические дополнения.

И заключительный совет – после таких примеров очень полезна гимнастика для глаз и какая-нибудь хорошая музыка для релаксации =)

Желаю успехов!

Решения и ответы:

Пример 3: Решение: запишем расширенную матрицу системы и с помощью элементарных преобразований получим базисное решение:
 
(1) Первую и вторую строки поменяли местами.

(2) Ко второй строке прибавили первую строку, умноженную на –2. К третьей строке прибавили первую строку, умноженную на 5.
(3) Третью строку разделили на 3.
(4) К третьей строке прибавили вторую строку, умноженную на 2.
(5) Третью строку разделили на 7.
(6) Наименьшее кратное чисел 3-го столбца (–3, 5, 1) равно 15-ти. Первую строку умножили на 5, вторую строку умножили на –3, третью строку умножили на 15.
(7) К первой строке прибавили 3-ю строку. Ко второй строке прибавили 3-ю строку.
(8) Первую строку разделили на 5, вторую строку разделили на –3, третью строку разделили на 15.
(9) Наименьшее кратное ненулевых чисел 2-го столбца (–2 и 1) равно: 2. Вторую строку умножили на 2
(10) К первой строке прибавили вторую строку.
(11) Вторую строку разделили на 2.
Выразим базисные переменные  через свободные переменные :

Ответ: общее решение: 

Пример 5: Решение: обратную матрицу найдём с помощью элементарных преобразований:

(1) Первую строку умножили на –15, вторую строку умножили на 3, третью строку умножили на 5.

(2) Ко 2-ой и 3-ей строкам прибавили первую строку.
(3) Первую строку разделили на –15, вторую строку разделили на –3, третью строку разделили на –5.
(4) Вторую строку умножили на 7, третью строку умножили на –9.
(5) К третьей строке прибавили вторую строку.

(6) Вторую строку разделили на 7.

(7) Первую строку умножили на 27, вторую строку умножили на 6, третью строку умножили на –4.
(8) К первой и второй строкам прибавили третью строку.
(9) Третью строку разделили на –4. К первой строке прибавили вторую строку, умноженную на –1.
(10) Вторую строку разделили на 2.
(11) Каждую строку разделили на 27.
В результате: 
Ответ

НОУ ИНТУИТ | Лекция | Задачи линейной алгебры

Аннотация: Познакомимся с инструментами Octave, предназначенными для работы с векторами и матрицами, а также с возможностями, которые предоставляет пакет при непосредственном решении задач линейной алгебры.

5.1 Ввод и формирование векторов и матриц

Векторы и матрицы в Octave задаются путём ввода их элементов. Элементы вектора-строки отделяют пробелами или запятыми, а всю конструкцию заключают в квадратные скобки:

	
>>> a =[2 -3 5 6 -1 0 7 -9]
a = 2 -3 5 6 -1 0 7 -9
>>> b =[ -1,0,1]
b = -1 0 1

Вектор-столбец можно задать, если элементы отделять друг от друга точкой с запятой:

	
>>> c=[-pi; -pi / 2; 0; pi / 2; pi ]
c =
-3.14159
-1.57080
0.00000
1.57080
3.14159

Обратиться к элементу вектора можно указав имя вектора, а в круглых скобках — номер элемента, под которым он хранится в этом векторе:

	
>>> a( 1 )
ans = 2
>>> b( 3 )
ans = 1
>>> c( 5 )
ans = 3.1416

Ввод элементов матрицы также осуществляется в квадратных скобках, при этом элементы строки отделяются друг от друга пробелом или запятой, а строки разделяются между собой точкой с запятой:

	
>>> Matr=[0 1 2 3; 4 5 6 7 ]
Matr =
0 1 2 3
4 5 6 7

Обратиться к элементу матрицы можно указав после имени матрицы, в круглых скобках, через запятую, номер строки и номер столбца, на пересечении которых элемент расположен:

	
>>> Matr ( 2, 3 )
ans = 6
>>> Matr ( 1, 1 )
ans = 0
>>> Matr ( 1, 1 )=pi; Matr ( 2, 4 )= _pi;
>>> Matr
Matr =
3.1416 1.0000 2.0000 3.0000
4.0000 5.0000 6.0000 -3.1416

Матрицы и векторы можно формировать, составляя их из ранее заданных матриц и векторов:

	
>>> a=[-3 0 2 ]; b=[3 2 -1]; c =[5 -2 0 ];
>>> M=[a b c ] % Горизонтальная конкатенация векторов–строк
M = -3 0 2 3 2 -1 5 -2 0 % результат — вектор–строка
>>> N=[a; b; c ] % Вертикальная конкатенация векторов–строк,
% результат — матрица
N =
	-3  0  2
	 3  2 -1
	 5 -2  0
>>> Matrica =[N N N] % Горизонтальная конкатенация матриц
Matrica =
	-3  0  2 -3  0  2 -3  0  2
	 3  2 -1  3  2 -1  3  2 -1
	 5 -2  0  5 -2  0  5 -2  0
>>> Tablica =[M;M;M] % Вертикальная конкатенация матриц
Tablica =
	-3 0 2 3 2 -1 5 -2 0
	-3 0 2 3 2 -1 5 -2 0
	-3 0 2 3 2 -1 5 -2 0

Важную роль при работе с матрицами играет знак двоеточия «:». Примеры с подробными комментариями приведены в листинге 5.1.

	
>>> Tabl =[ -1.2 3.4 0.8; 0.9 -0.1 1.1; 7.6 -4.5 5.6; 9.0 1.3 -8.5]
Tabl =
	-1.20000 3.40000 0.80000
	0.90000 -0.10000 1.10000
	7.60000 -4.50000 5.60000
	9.00000 1.30000 -8.50000
>>> Tabl( :, 3 ) % Выделить из матрицы 3-й столбец
ans =
	0.80000
	1.10000
	5.60000
	-8.50000
>>> Tabl( 1, : ) % Выделить из матрицы 1-ю строку
ans = -1.20000 3.40000 0.80000
>>> Matr=Tabl( 2 : 3, 1 : 2 ) % Выделить из матрицы подматрицу
Matr =
0.90000 -0.10000
7.60000 -4.50000
% Вставить подматрицу в правый нижний угол исходной матрицы
>>> Tabl( 3 : 4, 2 : 3 )=Matr
Tabl =
	-1.20000 3.40000  0.80000
	0.90000  -0.10000 1.10000
	7.60000  0.90000  -0.10000
	9.00000  7.60000  -4.50000
>>> Tabl( :, 2 ) = [ ] % Удалить из матрицы 2-й столбец
Tabl =
	-1.20000 0.80000
	0.90000  1.10000
	7.60000  -0.10000
	9.00000  -4.50000
>>> Tabl( 2, : ) = [ ] % Удалить из матрицы 2-ю строку
Tabl =
	-1.20000 0.80000
	7.60000 -0.10000
	9.00000 -4.50000
>>> Matr % Представить матрицу в виде вектора–столбца
Matr =
	0.90000 -0.10000
	7.60000 -4.50000
>>> Vector=Matr ( : )
Vector =
	0.90000
	7.60000
	-0.10000
	-4.50000
>>> V=Vector( 1 : 3 ) % Выделить из вектора элементы со 1-го по 3-й
V =
	0.90000
	7.60000
	-0.10000
>>> V( 2 ) = [ ] % Удалить из массива 2-й элемент
V =
	0.90000
	-0.10000
Листинг 5.1. Пример использования знака двоеточия «:» {-1} = I $$$

, где $$ I $$ — единичная матрица, все элементы которой равны нулю, за исключением элементов на главной диагонали, которые равны $$ 1 $$.

Пусть будет матрица:

$$$ A = \ left (\ begin {array} {ccc} 1 & 1 & 0 \\ 1 & 0 & 1 \\ 0 & 1 & 0 \ end {array} \ справа) $$$

Как найти обратную матрицу, используя метод исключения Гаусса?

1) Единичная матрица добавляется к матрице $$ A $$.

$$$ \ left (\ begin {array} {cccccc} 1 & 1 & 0 & 1 & 0 & 0 \\ 1 & 0 & 1 & 0 & 1 & 0 \\ 0 & 1 & 0 & 0 & 0 & 1 \ end {array} \ right) $$$

(Обратите внимание, что $$ I = \ left (\ begin {array} {ccc} 1 & 0 & 0 \\ 0 & 1 & 0 \\ 0 & 0 & 1 \ end {array} \ right) $$)

2) С помощью метода Гаусса мы попытаемся передать единичную матрицу в левую часть.{-1} $$.

Что является инверсией следующей матрицы?

$$$ A = \ left (\ begin {array} {ccc} 1 & 1 & 0 \\ 1 & 0 & 1 \\ 0 & 1 & 0 \ end {array} \ right) $$$

Мы должны следовать процедуре шаг за шагом.

1) Прежде всего, единичная матрица добавляется справа от исходной матрицы:

$$$ \ left (\ begin {array} {cccccc} 1 & 1 & 0 & 1 & 0 & 0 \\ 1 & 0 & 1 & 0 & 1 & 0 \\ 0 & 1 & 0 & 0 & 0 & 1 \ end {array} \ right) $$$

2) Мы должны «сдвинуть» единичную матрицу влево с помощью метода Гаусса.

Этот метод требует некоторой интуиции, поскольку он не является точным руководством. В любом случае интуицию можно заменить практикой, и метод Гаусса оказывается намного проще, чем кажется на первый взгляд.

$$$ \ left (\ begin {array} {cccccc} 1 & 1 & 0 & 1 & 0 & 0 \\ 1 & 0 & 1 & 0 & 1 & 0 \\ 0 & 1 & 0 & 0 & 0 & 1 \ end {array} \ right) \ rightarrow (row2-row1) \ rightarrow \ left (\ begin {array} {cccccc} 1 & 1 & 0 & 1 & 0 & 0 \\ 0 & -1 & 1 & -1 & 1 & 0 \\ 0 & 1 & 0 & 0 & 0 & 1 \ end {array} \ right) $$$

$$$ \ left (\ begin {array} {cccccc} 1 & 1 & 0 & 1 & 0 & 0 \\ 0 & -1 & 1 & -1 & 1 & 0 \\ 0 & 1 & 0 & 0 & 0 & 1 \ end {array} \ right) \ rightarrow (row3 + row2 ) \ rightarrow \ left (\ begin {array} {cccccc} 1 & 1 & 0 & 1 & 0 & 0 \\ 0 & -1 & 1 & -1 & 1 & 0 \\ 0 & 0 & 1 & -1 & 1 & 1 \ end {array} \ right) $$$

$$$ \ left (\ begin {array} {cccccc} 1 & 1 & 0 & 1 & 0 & 0 \\ 0 & -1 & 1 & -1 & 1 & 0 \\ 0 & 0 & 1 & -1 & 1 & 1 \ end {array} \ right) \ rightarrow (row2-row3) \ rightarrow \ left (\ begin {array} {cccccc} 1 & 1 & 0 & 1 & 0 & 0 \\ 0 & -1 & 0 & 0 & 0 & -1 \\ 0 & 0 & 1 & -1 & 1 & 1 \ end {array} \ right) $$$

$$$ \ left (\ begin {array} {cccccc} 1 & 1 & 0 & 1 & 0 & 0 \\ 0 & -1 & 0 & 0 & 0 & -1 \\ 0 & 0 & 1 & -1 & 1 & 1 \ end {array} \ right) \ rightarrow (row1 + row2) \ rightarrow \ left (\ begin {array} {cccccc} 1 & 0 & 0 & 1 & 0 & -1 \\ 0 & -1 & 0 & 0 & 0 & -1 \\ 0 & 0 & 1 & -1 & 1 & 1 \ end {array} \ right) $$$

Наконец, $$ row2 $$ умножается на $$ (- 1 ) $$, и поэтому мы имеем идентичность слева. {- 1} $.

Обратная матрица 2 $ \ times $ 2

Пример 1: Найдите обратное

$ A = \ left [{\ begin {array} {* {20} {c}} 1 и 3 \\ 2 и 7 \ end {array}} \ right]

долл. США

Решение:

Шаг 1: Присоедините единичную матрицу к правой части $ A $:

$ A = \ left [{\ begin {array} {* {20} {c}} 1 и 3 \\ 2 и 7 \ end {array} \ left | {\ begin {array} {* {20} {c}} \ color {blue} {1} & \ color {blue} {0} \\ \ color {синий} {0} & \ color {синий} {1} \ end {array}} \ right.} \верно] $

Шаг 2: Применяйте к этой матрице строковые операции, пока левая часть не уменьшится до $ I $. Вычисления:

$$ \ begin {выровнено} & \ left [{\ begin {array} {* {20} {c}} \ color {красный} {1} & \ color {красный} {3} \\ {2 — \ color {blue} {2} \ cdot \ color {red} {1}} & {7 — \ color {blue} {2} \ cdot \ color {red} {3}} \ end {array} \ left | {\ begin {array} {* {20} {c}} \ цвет {красный} {1} & \ цвет {красный} {0} \\ {0 — \ color {blue} {2} \ cdot \ color {red} {1}} & {1 — \ color {blue} {2} \ cdot \ color {red} {0}} \ end {array}} \ right.} \ right] \ \ Row2 = Row2 — \ color {blue} {2} \ cdot \ color {red} {Row1} \\ & \ left [{\ begin {array} {* {20} {c}} 1 и 3 \\ 0 и 1 \ end {array} \ left | {\ begin {array} {* {20} {c}} 1 & 0 \\ {- 2} & 1 \ end {array}} \ right.} \ right] \\ & \ left [{\ begin {array} {* {20} {c}} {1 — \ color {blue} {3} \ cdot \ color {red} {0}} & {3 — \ color {blue} {3} \ cdot \ color {red} {1}} \\ \ color {красный} {0} & \ color {красный} {1} \ end {array} \ left | {\ begin {array} {* {20} {c}} {1 — \ color {blue} {3} \ cdot \ color {red} {(- 2)}} & {0 — \ color {blue} {3} \ cdot \ color {red} {1}} \\ \ color {красный} {- 2} & \ color {красный} {1} \ end {array}} \ right.{-1} = \ left [{\ begin {array} {* {20} {c}} 7 & {- 3} \\ {-2} & 1 \ end {array}} \ right] $

Необратимая матрица

Если $ A $ необратимо , то слева появится нулевая строка.

Пример 2: Найдите обратное

$ A = \ left [{\ begin {array} {* {20} {c}} 1 & {- 3} \\ {- 2} и 6 \ end {array}} \ right] $

Решение:

Шаг 1: Присоедините единичную матрицу к правой стороне A:

$ \ left [{\ begin {array} {* {20} {c}} 1 & {- 3} \\ {- 2} и 6 \ end {array} \ left | {\ begin {array} {* {20} {c}} \ color {blue} {1} & \ color {blue} {0} \\ \ color {синий} {0} & \ color {синий} {1} \ end {array}} \ right.} \верно] $

Шаг 2: Применение операций со строками

$ \ left [{\ begin {array} {* {20} {c}} \ color {красный} {1} & \ color {красный} {- 3} \\ {- 2 + \ color {blue} {2} \ cdot \ color {red} {1}} & {6 + \ color {blue} {2} \ cdot \ color {red} {(- 3)}} \ end {array} \ left | {\ begin {array} {* {20} {c}} \ цвет {красный} {1} & \ цвет {красный} {0} \\ {0 + \ color {blue} {2} \ cdot \ color {red} {1}} & {1 + \ color {blue} {2} \ cdot \ color {red} {0}} \ end {array}} \ right.} \ right] Row2 = Row2 + \ color {red} {2} \ cdot \ color {blue} {Row1} $

$ \ left [{\ begin {array} {* {20} {c}} 1 & {- 3} \\ \ color {красный} {0} & \ color {красный} {0} \ end {array} \ left | {\ begin {array} {* {20} {c}} 1 & 0 \\ 0 и 1 \ end {array}} \ right.} \ right] _ {\ color {red} {\ leftarrow ZERO \ \ ROW}} $

Шаг 3: Вывод: Эта матрица необратима.

Обратная матрица 3 $ \ times $ 3

Пример 1: Найдите обратное

$ A = \ left [{\ begin {array} {* {20} {c}} 1 и 2 и 3 \\ 2 и 5 и 3 \\ 1 и 0 и 8 \ end {array}} \ right] $

Решение:

Шаг 1: Присоедините единичную матрицу к правой стороне A:

$ \ left [{\ begin {array} {* {20} {c}} 1 и 2 и 3 \\ 2 и 5 и 3 \\ 1 и 0 и 8 \ end {array} \ left | {\ begin {array} {* {20} {c}} 1 & 0 & 0 \\ 0 & 1 & 0 \\ 0 и 0 и 1 \ end {array}} \ right. {R2 = R2 — \ color {синий} {2} \ cdot R1} \ left [{\ begin {array} {* {20} {c}} 1 и 2 и 3 \\ {2 — \ color {blue} {2} \ cdot 1} & {5 — \ color {blue} {2} \ cdot 2} & {3 — \ color {blue} {2} \ cdot 3} \\ {1 \ color {red} {-} 1} & {0 \ color {red} {-} 2} & {8 \ color {red} {-} 3} \ end {array} \ left | {\ begin {array} {* {20} {c}} 1 & 0 & 0 \\ {0 — \ color {blue} {2} \ cdot 1} & {1 — \ color {blue} {2} \ cdot 0} & {0 — \ color {blue} {2} \ cdot 0} \\ {0 \ color {red} {-} 1} & {0 \ color {red} {-} 0} & {1 \ color {red} {-} 0} \ end {array}} \ right.{} \ left [{\ begin {array} {* {20} {c}} 1 и 2 и 3 \\ 0 & 1 & {- 3} \\ {0 \ color {blue} {+ 2} \ cdot 0} & {- 2 \ color {blue} {+ 2} \ cdot 1} & {5 \ color {blue} {+ 2} \ cdot (- 3) } \ end {array} \ left | {\ begin {array} {* {20} {c}} 1 & 0 & 0 \\ {- 2} & 1 & 0 \\ {- 1 \ color {blue} {+ 2} \ cdot (- 2)} & {0 \ color {blue} {+ 2} \ cdot 1} & {1 \ color {blue} {+ 2} \ cdot 0 } \ end {array}} \ right.} \ right] $

$ \ left [{\ begin {array} {* {20} {c}} 1 и 2 и 3 \\ 0 & 1 & {- 3} \\ 0 & 0 & {- 1} \ end {array} \ left | {\ begin {array} {* {20} {c}} 1 & 0 & 0 \\ {- 2} & 1 & 0 \\ {- 5} & 2 & 1 \ end {array}} \ right.{} \ left [{\ begin {array} {* {20} {c}} 1 и 2 и 3 \\ 0 & 1 & {- 3} \\ 0 и 0 и 1 \ end {array} \ left | {\ begin {array} {* {20} {c}} 1 & 0 & 0 \\ {- 2} & 1 & 0 \\ 5 & ​​{- 2} & {- 1} \ end {array}} \ right.} \ right] $

$ \ left [{\ begin {array} {* {20} {c}} 1 и 2 и 3 \\ 0 & 1 & {- 3} \\ 0 и 0 и 1 \ end {array} \ left | {\ begin {array} {* {20} {c}} 1 & 0 & 0 \\ {- 2} & 1 & 0 \\ {- 5} & {- 2} & 1 \ end {array}} \ right.{R1 = R1 — 3 \ cdot R3} \ left [{\ begin {array} {* {20} {c}} 1 и 2 и 0 \\ 0 & 1 & 0 \\ 0 и 0 и 1 \ end {array} \ left | {\ begin {array} {* {20} {c}} {- 14} & 6 и 3 \\ {13} & {- 5} & {- 3} \\ 5 & ​​{- 2} & {- 1} \ end {array}} \ right.} \ right] $

$ \ left [{\ begin {array} {* {20} {c}} 1 и 2 и 0 \\ 0 & 1 & 0 \\ 0 и 0 и 1 \ end {array} \ left | {\ begin {array} {* {20} {c}} {- 14} & 6 и 3 \\ {13} & {- 5} & {- 3} \\ 5 & ​​{- 2} & {- 1} \ end {array}} \ right.{- 1} = \ left [{\ begin {array} {* {20} {c}} {- 40} & {16} & 9 \\ {13} & {- 5} & {- 3} \\ 5 & ​​{- 2} & {- 1} \ end {array}} \ right] $

Матричный обратный метод

Матричный обратный метод Матричный обратный метод
для решения системы уравнений

См. Аналогичное обсуждение в нашем тексте, Рольф, на страницах 165-167 ;
нижеследующее продолжает предыдущее обсуждение.

Начнем с системы уравнений (справа),
быть решенным. Количество переменных должно равняться
. количество уравнений, здесь оба равны 3:
x y + 3z = 2
2x + y + 2z = 2
-2x 2 года + z = 3

О проф.M c Испытания Фарланда, вас спросят для решения вышеуказанной проблемы

«методом обратной матрицы» в три отдельных этапа, как показано ниже :

[1]
Запишите данную систему (см. Выше) в виде единой матрицы уравнение :
и nbsp
Переменные с заглавными буквами представляют матрицы (не числа), которые находятся прямо над ними.Следовательно Вышеприведенное уравнение является матричным уравнением. Обратите внимание, как элементы матрицы соответствуют числа в исходной системе трех уравнений
[2]
Решите матричное уравнение, полученное в шаг [1] выше; т.е. найти X.
В МАТРИЧНОМ ОБРАТНОМ МЕТОДЕ (в отличие от Гаусса / Джордана), мы решаем матричную переменную X как левое умножение обеих сторон указанной выше матрицы уравнение (AX = B) по A -1 .Обычно A -1 рассчитывается как отдельное упражнение ; в противном случае мы должны остановиться здесь, чтобы вычислить A -1 .
Левая часть (вверху) легко вычисляется
потому что единичная матрица я Появляется 3 .
Примечание: A -1 находится СЛЕВА. сторона обоих продуктов.
Шаг [2] завершается поиском А -1. B ===>
[3]
Используя [2] выше, запишите решение в исходная система :
оригинальная система
x y + 3z = 2
2x + y + 2z = 2
-2x 2 года + z = 3
решение исходной системы

(PDF) Обратная матрица с использованием метода исключения Гаусса по OpenMP

46 Обратная матрица с использованием метода исключения Гаусса по OpenMP

Авторские права © 2016 MECS I.J. Информационные технологии и информатика, 2016, 2, 41-46

ПОДТВЕРЖДЕНИЕ

Мы хотим поблагодарить наши семьи и друзей за поддержку и заботу

. Оба автора внесли равный вклад в

этой работы.

ССЫЛКИ

[1] Murthy, K.N.B. и C.S.R. Murthy, Gaussian-

Алгоритм на основе исключения

для решения линейных уравнений

на процессорах, подключенных к сетке. Компьютеры и цифровые технологии

, Труды IEE -, 1996.143 (6): с. 407-412.

[2] Алланде, С. и др., Модель производительности для OpenMP

приложений с привязкой к памяти в системах с несколькими сокетами.

Процедуры информатики, 2014. 29 (0): с. 2208-2218.

[3] Park, I. and S.W. Ким, Исследование приложений OpenMP на

программной распределенной системе с общей памятью на основе InfiniBand

. Параллельные вычисления, 2005. 31 (10–12): с. 1099-

1113.

[4] Guo, X., et al., Разработка масштабируемой гибридной неструктурированной конечно-элементной модели

MPI / OpenMP.

Компьютеры и жидкости, 2015. 110 (0): с. 227-234.

[5] Чжан С. и др. Параллельное вычисление модели потока прорыва плотины

с использованием OpenMP на многоядерном компьютере.

Journal of Hydrology, 2014. 512 (0): с. 126-133.

[6] Маронджиу А., П. Берджио и Л. Бенини, Поддержка

OpenMP на мультикластерном встроенном MPSoC.

Микропроцессоры и микросистемы, 2011. 35 (8): с. 668-

682.

[7] Jeun, W.-C., et al., Преодоление узких мест производительности

при использовании OpenMP на кластерах SMP. Parallel Computing,

2008. 34 (10): p. 570-592.

[8] Дорудиан С., Н. Гэмиан и М. Шарифи. Оценка

накладных расходов директив OpenMP. в области электротехники

Engineering (ICEE), 19-я иранская конференция 2011 г.

2011.

[9] Цзянь Г., Ю. Су и Дж. Цзянь-Мин, OpenMP-CUDA

Реализация многоуровневого быстрого многополюсного алгоритма

для электромагнитного моделирования в системах с несколькими графическими процессорами

. Антенны и распространение, IEEE Transactions

on, 2013. 61 (7): p. 3607-3616.

[10] Барлас, Г., Глава 4 — Программирование с общей памятью:

OpenMP, в многоядерном программировании и программировании на GPU, Г. Барлас,

Editor. 2015, Морган Кауфманн: Бостон.п. 165-238.

[11] Shengfei, L., et al. Оценка производительности

многопоточного умножения разреженной матрицы на вектор с использованием

OpenMP. в области высокопроизводительных вычислений и

коммуникаций, 2009 г. HPCC ’09. 11-я Международная конференция IEEE

по. 2009.

[12] Цзянь-Цзюнь, Х. и Л. Цин-Хуа. Dynamic Power-Aware

Алгоритмы планирования для наборов задач в реальном времени с отказоустойчивостью

в среде параллельных и распределенных вычислений

.по параллельной и распределенной обработке

Симпозиум

, 2005. Труды. 19-я Международная конференция IEEE.

2005.

[13] С.Ф. Макгинн и Р. Шоу. Параллельное устранение Gaussian

с использованием OpenMP и MPI. В Proceedings of

the International Symposium on High Performance

Computing Systems and Applications, 2002.

[14] Sibai, Fadi N., 2013. Моделирование и анализ производительности

параллельного исключения Гаусса на многоядерных компьютерах.

Журнал Университета Короля Сауда — Компьютеры и

Информационные науки. Elsevier, Vol. 26. С. 41–54.

[15] Михайлидис, П. Д., Маргаритис, К. Г. (2011). Parallel

прямые методы решения системы линейных уравнений

с конвейерной обработкой на многоядерности с использованием OpenMP. Журнал

Вычислительная и прикладная математика, 236 (3): 326–341.

Профили авторов

Юсеф С. Алсенани родился в Саудовской Аравии.

Алсенани получил степень магистра в области

Advanced Computer Science в

California Lutheran University, Thousand

Oaks, США, 2013. Он читает лекции в

University King Abdualziz, Саудовская Аравия.

Сейчас он аспирант Южного

Университета штата Иллинойс Карбондейл, США. Его текущий исследовательский интерес

— облачные вычисления.

Мадини О. Алассафи родился в Саудовской Аравии.

Алассафи получил степень магистра в области

Advanced Computer Science в

California Lutheran University, Thousand

Oaks, USA, 2013.Он читает лекции в

Университете короля Абдуалзиза, Саудовская Аравия.

Сейчас он аспирант Саутгемптонского университета

, Великобритания. Его текущее исследование

интересует облачные вычисления.

Устранение Гаусса Джордана — объяснение и примеры

Метод исключения Гаусса-Жордана — это алгоритм для решения линейной системы уравнений. Мы также можем использовать его, чтобы найти обратную матрицу. Давайте сначала посмотрим на определение:

Исключение Гаусса Джордана или Гаусса исключение — это алгоритм для решения системы линейных уравнений, представляющий ее в виде расширенной матрицы, сокращая ее с помощью операций со строками и выражая систему в сокращенной строке. -эшелонированная форма для нахождения значений переменных.

В этом уроке мы увидим детали метода исключения Гаусса и того, как решить систему линейных уравнений с использованием метода исключения Гаусса-Жордана. Примеры и практические вопросы будут приведены ниже.

Что такое метод исключения Гаусса?

Метод исключения Гаусса — это структурированный метод решения системы линейных уравнений. Таким образом, это алгоритм, и его можно легко запрограммировать для решения системы линейных уравнений. Основная цель метода исключения Гаусса-Джордана:

  • представить систему линейных уравнений в форме расширенной матрицы
  • затем выполнить операции строки $ 3 $ до тех пор, пока не будет получена сокращенная форма эшелона строк (RREF) достигнуто
  • Наконец, мы можем легко распознать решения из RREF

Давайте посмотрим, что такое расширенная матричная форма, операции со строками в $ 3 $, которые мы можем выполнять с матрицей, и уменьшенная форма эшелона строк матрицы.

Расширенная матрица

Система линейных уравнений показана ниже:

$ \ begin {align *} 2x + 3y & = \, 7 \\ x — y & = 4 \ end {align *} $

We запишет расширенную матрицу этой системы, используя коэффициенты уравнений и запишет ее в стиле , показанном ниже:

$ \ left [\ begin {array} {rr | r} 2 & 3 & 7 \\ 1 & -1 & 4 \ end {array} \ right] $

Пример использования одновременных уравнений $ 3 $ показан ниже:

$ \ begin {align *} 2x + y + z & = \, 10 \\ x + 2y + 3z & = 1 \\ — x — y — z & = 2 \ end {align *} $

Представление этой системы в виде расширенной матрицы:

$ \ left [\ begin {array} {rrr | r} 2 & 1 & 1 & 10 \\ 1 & 2 & 3 & 1 \\ — 1 & — 1 & — 1 & 2 \ end {array} \ right] $

Операции со строками в матрице

Есть $ 3 $ элементарных операций со строками , которые мы можем выполнять с матрицами.Это не изменит решения системы. Это:

  1. Обмен $ 2 $ строк
  2. Умножить строку на ненулевой ($ \ neq 0 $) скаляр
  3. Добавить или вычесть скалярное кратное одной строки из другой строки.

Форма сокращенного эшелона строк

Основная цель исключения Гаусса Джордана — использовать операции элементарной строки стоимостью 3 доллара в расширенной матрице, чтобы привести ее к форме сокращенного эшелона строк (RREF). Говорят, что матрица находится в сокращенной форме эшелона строк , также известной как каноническая форма строки , если выполняются следующие условия $ 4 $:

  1. Строки с нулевыми записями (все элементы этой строки равны $ 0 $). s) находятся внизу матрицы.
  2. Начальная запись (первая ненулевая запись в строке) каждой ненулевой строки соответствует справа ведущей записи строки непосредственно над ней.
  3. Начальная запись в любой ненулевой строке — $ 1 $.
  4. Все записи в столбце, содержащем начальную запись ($ 1 $), нулевые.

Как выполнить исключение Гаусса-Джордана

В методе исключения Гаусса-Джордана нет определенных шагов, но алгоритм ниже описывает шаги, которые мы выполняем, чтобы прийти к сокращенной форме эшелона строк расширенной матрицы.

  1. Поменяйте местами строки так, чтобы все строки с нулевыми записями находились внизу матрицы.
  2. Поменяйте местами строки так, чтобы строка с самой большой левой цифрой находилась наверху матрицы.
  3. Умножьте верхнюю строку на скаляр, который преобразует ведущую запись верхней строки в $ 1 $ (если ведущей записью верхней строки является $ a $, умножьте ее на $ \ frac {1} {a} $, чтобы получить $ 1 $).
  4. Добавьте или вычтите значения, кратные верхней строке, из других строк, чтобы все записи в столбце ведущей записи верхней строки были нулями.
  5. Выполните шаги $ 2 — 4 $ для следующей крайней левой ненулевой записи до тех пор, пока все ведущие записи каждой строки не станут $ 1 $.
  6. Поменяйте местами строки так, чтобы ведущая запись каждой ненулевой строки находилась справа от ведущей записи строки непосредственно над ней

На первый взгляд, запомнить / запомнить шаги не так просто. Это вопрос решения нескольких проблем, пока вы не освоитесь с процессом. Существует также фактор интуиции , который играет B-I-G роль в выполнении исключения Гаусса Джордана.

Давайте рассмотрим несколько примеров, чтобы пояснить процесс решения системы линейных уравнений с помощью метода исключения Гаусса-Джордана .

Пример 1

Решите систему, показанную ниже, используя метод исключения Гаусса Джордана:

$ \ begin {align *} {- x} + 2y & = \, {- 6} \\ { 3x} — 4y & = {14} \ end {align *} $

Решение

Первый шаг — написать расширенную матрицу системы.Мы показываем это ниже:

$ \ left [\ begin {array} {r r | r} — 1 & 2 & — 6 \\ 3 & -4 & 14 \ end {array} \ right] $

Теперь наша задача состоит в том, чтобы преобразовать матрицу в сокращенную форму эшелона строк (RREF), выполнив $ 3 $ элементарные операции со строками.

У нас есть расширенная матрица:

$ \ left [\ begin {array} {r r | r} — 1 & 2 & — 6 \\ 3 & — 4 & 14 \ end {array} \ right] $

Шаг 1:

Мы можем умножить первую строку на $ — 1 $, чтобы получить ведущий вход $ 1 $.Показано ниже:

$ \ left [\ begin {array} {r r | r} 1 & — 2 & 6 \\ 3 & — 4 & 14 \ end {array} \ right] $

Шаг 2:

Теперь мы можем умножить первую строку на $ 3 $ и вычесть ее из второй ряд. Показано ниже:

$ \ left [\ begin {array} {r r | r} 1 & -2 & 6 \\ {3 — (1 \ times 3)} & {-4 — (-2 \ times 3)} & {14 — (6 \ times 3)} \ end {array} \ справа] $

$ = \ left [\ begin {array} {rr | r} 1 & — 2 & 6 \\ 0 & 2 & — 4 \ end {array} \ right] $

У нас есть $ 0 $ как первая запись во второй строке.

Шаг 3:

Чтобы сделать вторую запись второй строки $ 1 $, мы можем умножить вторую строку на $ \ frac {1} {2} $. Показано ниже:

$ \ left [\ begin {array} {r r | r} 1 & — 2 & 6 \\ {\ frac {1} {2} \ times 0} & {\ frac {1} {2} \ times 2} & {\ frac {1} {2} \ times — 4} \ end {array} \ right] $

$ = \ left [\ begin {array} {rr | r} 1 & — 2 & 6 \\ 0 & 1 & — 2 \ end {array} \ right] $

Шаг 4:

Мы почти у цели!

Вторая запись первой строки должна быть $ 0 $.Для этого мы умножаем вторую строку на $ 2 $ и добавляем ее к первой строке. Показано ниже:

$ \ left [\ begin {array} {r r | r} {1 + (0 \ times 2)} & {- 2 + (1 \ times 2)} & {6 + (- 2 \ times 2)} \\ 0 & 1 & — 2 \ end {array} \ справа] $

$ = \ left [\ begin {array} {rr | r} 1 & 0 & 2 \\ 0 & 1 & — 2 \ end {array} \ right] $

Это сокращенный эшелон строки , форма . Из расширенной матрицы мы можем написать два уравнения (решения):

$ \ begin {align *} x + 0y & = \, 2 \\ 0x + y & = -2 \ end {align *} $

$ \ begin {align *} x & = \, 2 \\ y & = — 2 \ end {align *} $

Таким образом, решение системы уравнений: $ x = 2 $ и $ y = — 2 $.

Пример 2

Решите систему, показанную ниже, используя метод исключения Гаусса Джордана:

$ \ begin {align *} x + 2y & = \, 4 \\ x — 2y & = 6 \ end { align *} $


Решение

Запишем расширенную матрицу системы уравнений:

$ \ left [\ begin {array} {rr | r} 1 & 2 & 4 \\ 1 & — 2 & 6 \ end {array} \ right] $

Теперь мы выполняем элементарные операции со строками с этой матрицей, пока не получим сокращенную форму эшелона строк.

Шаг 1:

Мы умножаем первую строку на $ 1 $, а затем вычитаем ее из второй строки. Это в основном вычитание первой строки из второй:

$ \ left [\ begin {array} {r r | r} 1 & 2 & 4 \\ 1 — 1 & — 2 — 2 & 6 — 4 \ end {array} \ right] $

$ = \ left [\ begin {array} {r r | r} 1 & 2 & 4 \\ 0 & — 4 & 2 \ end {array} \ right] $

Шаг 2:

Мы умножаем вторую строку на $ — \ frac {1} {4} $, чтобы получить вторая запись строки, $ 1 $:

$ \ left [\ begin {array} {rr | r} 1 и 2 и 4 \\ 0 \ times — \ frac {1} {4} & — 4 \ times — \ frac {1} {4} и 2 \ times — \ frac {1} {4} \ end {массив} \ right] $

$ = \ left [\ begin {array} {rr | r} 1 & 2 & 4 \\ 0 & 1 & — \ frac {1} {2} \ end {array} \ right] $

Шаг 3:

Наконец, мы умножаем вторую строку на $ — 2 $ и добавьте его в первую строку, чтобы получить уменьшенную форму эшелона строк этой матрицы:

$ \ left [\ begin {array} {rr | r} 1 + (- 2 \ times 0) & 2+ (- 2 \ times 1) & 4 + (- 2 \ times — \ frac {1} {2}) \\ 0 & 1 & — \ frac {1 } {2} \ end {array} \ right] $

$ = \ left [\ begin {array} {rr | r} 1 & 0 & 5 \\ 0 & 1 & — \ frac {1} {2} \ end {array} \ right] $

Это сокращенный эшелон строки , форма .Из расширенной матрицы мы можем написать два уравнения (решения):

$ \ begin {align *} x + 0y & = \, 5 \\ 0x + y & = — \ frac {1} {2} \ end {align *} $

$ \ begin {align *} x & = \, 5 \\ y & = — \ frac {1} {2} \ end {align *} $

Таким образом, решение системы уравнений составляет $ x = 5 $ и $ y = — \ frac {1} {2} $.

Практические вопросы
  1. Решите систему, показанную ниже, используя метод исключения Гаусса Джордана:

    $ \ begin {align *} 2x + y & = \, — 3 \\ — x — y & = 2 \ end {align *} $

  2. Решите систему, показанную ниже, используя метод исключения Гаусса Джордана:

    $ \ begin {align *} x + 5y & = \, 15 \\ — x + 5y & = 25 \ end {align *} $

Ответы

  1. Начнем с написания расширенной матрицы системы уравнений:

    $ \ left [\ begin {array} {rr | r} 2 & 1 & — 3 \\ — 1 & — 1 & 2 \ end {array} \ right] $

    Теперь мы выполняем элементарные операции со строками, чтобы прийти к нашему решению.

    Первый,
    Меняем знаки второй строки местами. Итак, имеем:
    $ \ left [\ begin {array} {r r | r} 1 & 1 & — 2 \\ 2 & 1 & — 3 \ end {array} \ right] $
    Во-вторых,
    Мы дважды вычитаем первую строку из второй строки:
    $ \ left [\ begin {array} { rr | r} 1 & 1 & — 2 \\ 2 — (2 \ times 1) & 1 — (2 \ times 1) & — 3 — (2 \ times — 2) \ end {array} \ right] $
    $ = \ left [\ begin {array} {rr | r} 1 & 1 & — 2 \\ 0 & — 1 & 1 \ end {array} \ right] $
    В-третьих,
    Мы инвертируем вторую строку, чтобы получить:
    $ = \ left [\ begin {array} {rr | r} 1 & 1 & — 2 \\ 0 & 1 & — 1 \ end {array} \ right] $
    Наконец,
    Мы вычитаем вторую строку из первой и получаем:
    $ = \ left [\ begin { массив} {rr | r} 1 & 0 & — 1 \\ 0 & 1 & — 1 \ end {array} \ right] $

    Из этой расширенной матрицы мы можем написать два уравнения (решения):

    $ \ begin {align *} x + 0y & = \, — 1 \\ 0x + y & = — 1 \ end {align *} $

    $ \ begin {align *} x & = \, — 1 \\ y & = — 1 \ end {align *} $

    Таким образом, решение системы уравнений: $ x = — 1 $ и $ y = — 1 $.

  2. Расширенная матрица системы:
    $ \ left [\ begin {array} {rr | r} 1 & 5 & 15 \\ — 1 & 5 & 25 \ end {array} \ right] $
    Давайте приведите эту матрицу к приведенной форме эшелона строк и найдите решение системы.

    Сначала
    Отмените первую строку, затем вычтите ее из второй строки, чтобы получить:
    $ \ left [\ begin {array} {rr | r} 1 & 5 & 15 \\ — 1 — (- 1) & 5 — (- 5) & 25 — (- 15) \ end {array} \ right] $
    $ = \ left [\ begin {array} {rr | r} 1 & 5 & 15 \\ 0 & 10 & 40 \ end {array} \ right] $
    Во-вторых,
    Разделите вторую строку на $ 10 $, чтобы получить:
    $ \ left [\ begin {array} {rr | r} 1 & 5 & 15 \\ 0 & 1 & 4 \ end {array} \ right] $
    Затем
    Умножьте вторую строку на $ 5 $ и вычтите ее из первой строки, чтобы окончательно получить решение:
    $ \ left [\ begin {array} {rr | r} 1 — (5 \ times 0) & 5 — (5 \ times 1) & 15 — (5 \ times 4) \\ 0 & 1 & 4 \ end {array} \ right] $
    $ = \ left [ \ begin {array} {rr | r} 1 & 0 & — 5 \\ 0 & 1 & 4 \ end {array} \ right] $
    Это сокращенная форма эшелона строк (RREF).Из этой расширенной матрицы мы можем написать два уравнения (решения):

    $ \ begin {align *} x & = \, — 5 \\ y & = 4 \ end {align *} $

    Таким образом, решение системы уравнений составляет $ x = — 5 $ и $ y = 4 $.

Предыдущий урок | Главная страница | Следующий урок

Метод исключения Гаусса, шаг за шагом

Благодаря более чем 3,4 миллионам загрузок, MATH 42 пользуется доверием во всем мире среди учащихся средних и старших классов, а также для решения математических задач, понимания решений и получения более высоких оценок.За небольшую часть стоимости частных репетиторов MATH 42 предлагает пошаговые решения, интеллектуальные подходы и A…

Частичный поворот: пример прямого исключения: шаг 2 Изучение значений первого столбца | -0.001 | и | 2,5 | или 0,0001 и 2,5. Наибольшее абсолютное значение равно 2,5, поэтому строка 2 заменяется строкой 3 Выполнение перестановки строк Частичное вращение: пример прямого исключения: шаг 2 Выполнение прямого исключения приводит к: частичному повороту …

Таким образом, результирующий Иногда решение Гаусса-Жордана необходимо улучшить, применяя простой численный метод, например метод простой итерации.Таким образом, решение превращается в двухэтапный: сначала применяется алгоритм Гаусса-Жордана, а затем численный метод, принимающий начальное решение в качестве решения на первом этапе. Практические задачи

в его умении набирать математику. Кроме того, ниже приводится пошаговое использование метода исключения Гаусса для решения системы уравнений. 2 Решение системы уравнений 2.1 Расширенная матрица Предположим, кто-то хочет решить следующую систему x + 2y + 3z = 6 2x y + 4z = 8 x + 8y + 2z = 12 (1) для x, y и z.

Шаг 1: Числа. Сгенерируйте случайные числа (максимум 10 000) из распределения Гаусса. Среднее значение распределения должно быть (пределы ± 1 000 000) и его стандартное отклонение (пределы ± 1 000 000). В числах должны быть значащие цифры (минимум 2, максимум 20).

Исключение Гаусса и Гаусса-Джордана. Существует два метода решения систем линейных уравнений: метод исключения Гаусса; Исключение Гаусса-Жордана; Оба они основаны на наблюдении, что системы уравнений эквивалентны, если они имеют один и тот же набор решений и выполняют простые операции со строками матрицы, известные как элементарные операции со строками или (ERO).

15 августа 2014 г. · Методом исключения Гаусса-Жордана мы приводим заданную матрицу к сокращенной форме эшелона строк. Я реализовал довольно много алгоритмов исключения Гаусса и исключения Гаусса-Жордана. Обычный алгоритм исключения Гуасса очень прост и может быть найден в [1]. В этом алгоритме есть две проблемы.

Шаг 1: Числа. Сгенерируйте случайные числа (максимум 10 000) из распределения Гаусса. Среднее значение распределения должно быть (пределы ± 1 000 000) и его стандартное отклонение (пределы ± 1 000 000).В числах должны быть значащие цифры (минимум 2, максимум 20).

Как еще называется метод исключения Гаусса? варианты ответа. Устранение. Замена. … Что делать дальше? варианты ответа. Mult. R2 на -3. Mult. R2 by 1/3

Калькулятор расширенной матричной системы уравнений

Калькулятор простой матрицы Это приведет к преобразованию матрицы размером до 5×6 в уменьшенную ступенчатую форму путем исключения Гаусса. Будет напечатана каждая операция элементарной строки. Если матрица меньше 5×6, поместите ее в верхний левый угол и оставьте дополнительные строки и столбцы пустыми.Были включены некоторые примерные значения. Преобразование систем линейных уравнений в матрицы. Каждое уравнение в системе становится строкой. Если матрица представляет собой расширенную матрицу, построенную из системы линейных уравнений, то эквивалентная по строке матрица будет иметь то же решение, что и исходная матрица. Если существует A-1 (обратная к A), мы можем умножить обе стороны на A-1, чтобы получить X = A-1 B. Чтобы решить эту систему линейных уравнений в Excel, выполните следующие шаги. В дополнение к отличным ответам @AMiT Kumar и @Scott, SymPy 1.0 добавил еще дополнительные функции. Для недоопределенной линейной системы уравнений я попытался заставить ее работать, не углубляясь в sympy.solvers.solveset. Решите систему уравнений методом исключения Калькулятор тенденции поиска: Галерея Краткое руководство по использованию линейных систем Узнайте, почему использование системного построения графиков будет иметь тенденцию в 2016 году, а также в 2015 году. Требуется больше изображений такого метода построения графиков систем, как этот, для 2016 года. для нелинейного метода каждая из них все еще работает. Найдите матрицу в сокращенной форме эшелона строк, которая является строкой, эквивалентной заданной матрице размера mxn A.Укажите размеры матрицы. Пожалуйста, выберите размер матрицы во всплывающих меню, затем нажмите кнопку «Отправить». Количество строк: m =. Калькулятор системы линейных уравнений — пошаговое решение системы линейных уравнений, исключение Гаусса, правило Крамера, метод обратной матрицы, анализ для Также вы можете вычислить ряд решений в системе линейных уравнений (проанализировать совместимость) с помощью теоремы Руше-Капелли. Расширенная матрица коэффициентов и метод исключения Гаусса могут быть использованы для упрощения процесса решения линейных систем.Чтобы решить систему с использованием матриц и исключения Гаусса, сначала используйте коэффициенты для создания расширенной матрицы. Примените операции с элементарными строками как средство для получения матрицы в форме верхнего треугольника. Напишите расширенную матрицу для следующей системы уравнений. Затем решите систему, используя сокращенную форму эшелона строк на графическом калькуляторе. 3x y 3z 2 2x y 2z 1 4x 2y 5z 5 Напишите расширенную матрицу и введите ее в калькулятор. Расширенная матрица B 2. Начните с ввода матрицы.КЛЮЧЕВЫЕ ДЕЙСТВИЯ: просмотрите матрицы на странице 163. В статье основное внимание уделяется использованию алгоритма для решения системы линейных уравнений. Мы будем иметь дело с матрицей коэффициентов. Метод исключения Гаусса не работает с сингулярными матрицами (они приводят к делению на ноль). Входные данные: для N неизвестных входные данные являются расширенными. 20 февраля 2017 г. · Как написать расширенную матрицу для системы линейных уравнений # 7x-5y + z = 13, 19x = 8z = 10 #? Предварительное вычисление матричных операций со строками Решение системы уравнений с использованием матрицы 1 Ответ На этой странице объясняется, как решать линейные системы, вычислять различные разложения, такие как LU, QR, SVD, собственные разложения… Базовое линейное решение. Проблема: у вас есть система уравнений, которую вы записали в виде единого матричного уравнения. Решите систему уравнений методом исключения Калькулятор тенденции поиска: Галерея Краткое прочтение о линейных системах. Узнайте, почему использование системного построения графиков будет иметь тенденцию в 2016 году, а также 2015 Требуется больше изображений системного метода построения графиков, подобного этому для 2016 года Цветное фото с нелинейным методом построения графиков Эта ссылка для нелинейного метода построения графиков все еще работает. Калькулятор сложной матрицы. Вычисление сложных матричных выражений и выполнение матричных операций с участием сложных матриц, решение систем сложных линейных уравнений.Вычислить детерминант, обратную, сопряженную, ранговую, сокращенную форму ряда комплексных матриц и работать со сложными расширенными матрицами, представляющими линейные системы уравнений, и сделать … Эта расширенная матрица представляет систему уравнений: она решается обратной подстановкой. Подстановка z = 3 во второе уравнение дает y = 5. Затем подстановка z = 3 и y = 5 в первое уравнение дает x = 7. Матрица A 2×2 называется матрицей коэффициентов системы уравнений. Часто это уравнение записывается как одна расширенная матрица: теперь мы можем использовать метод исключения Гаусса для решения этого матричного уравнения относительно x и y (в отличие от прямой подстановки одного уравнения в другое).С этой матрицей вы можете произвести ровно три операции: Поменять местами строки; Добавьте одну строку в другую; Умножьте каждый множитель одной строки на константу; Вы хотите получить треугольную матрицу. Таким образом, вы впоследствии исключаете одну переменную из системы уравнений, пока не получите такую ​​матрицу: 04 декабря 2020 г. · Когда использовать расширенную матрицу. Вы должны использовать расширенную матрицу, когда методы замены и исключения либо непрактичны, либо невозможны вместе. . Теперь давайте рассмотрим три разные системы и воспользуемся тем, что мы только что узнали, чтобы подумать, какой метод наиболее полезен для каждой системы.1) 5x — 58y = -883-5x + 2y = -13 Простой калькулятор матрицы. 5б. Обращение матрицы с помощью исключения Гаусса-Жордана. 6. Матрицы и линейные уравнения. Пример — электронное приложение системы уравнений 3 × 3. Найдите электрические токи, показанные путем решения матричного уравнения (полученного с использованием закона Кирхгофа), возникающего из этой схемы. Решатель линейных систем — это калькулятор линейных систем для линейных уравнений и матричного вычисления или для квадратных матриц. Он вычисляет собственные значения и собственные векторы в одной и той же форме, получая диагональную форму во всей этой симметричной матричной форме.Также он вычисляет обратные, транспонированные, собственные значения, LU-разложение квадратных матриц.

Решение систем уравнений с использованием матриц с использованием обратных матриц для оценки системы уравнений. (Используйте калькулятор) Пример: 3x — 2y + z = 24 2x + 2y + 2z = 12 x + 5y — 2z = -31 Это калькулятор, который поможет вам найти обратную матрицу 3 × 3. Попробуйте бесплатный калькулятор Mathway и средство решения задач ниже, чтобы попрактиковаться в различных математических темах …

Использование матриц для решения системы уравнений на TI-85           = + = + = 3x — 2y — 9z 9 2x 3y — z -2 x — yz 8 Решение: Чтобы решить, нам нужно ввести и назвать расширенную матрицу.          — — — — — 3 2 9 9 2 3 1 2 1 8

Использование матриц для решения систем линейных уравнений. Примечание: идеи этого урока может быть довольно сложно изложить словами. Видео поможет вам многое объяснить, так как в нем много наглядных диаграмм, показывающих, что происходит, шаг за шагом. Мы можем представить всю линейную систему с помощью расширенной матрицы:

d .. Какого размера матрица расширенных коэффициентов для системы из 3 уравнений с 4 переменными? е. Коммутативно ли сложение матриц вообще?

2 мая 2013 г. · Вопрос гласит: Решите следующие системы уравнений, используя операции со строками в расширенной матрице.Вопрос 1. x + y-z = 5 x + 2y-3z = 9 x-y + 3z = 3 Вопрос 2.

1.3. Решение систем линейных уравнений путем нахождения приведенной эшелонированной формы матрицы и обратной подстановки. Чтобы решить систему линейных уравнений, представленную матричным уравнением, мы сначала добавляем вектор правой части к матрице коэффициентов, чтобы сформировать расширенную матрицу коэффициентов.

Решение линейной системы включает решение матричного уравнения. Поскольку это матрица ×, то это набор случаев, когда система называется квадратной.Если уравнений больше, чем неизвестных, и система переопределена. Для второго примера ранг равен рангу расширенной матрицы.

13 марта 2020 г. · Чтобы решить систему уравнений с помощью графического калькулятора TI-83 или TI-84, систему уравнений необходимо поместить в расширенную матрицу. Расширенная матрица может быть введена в калькулятор, который преобразует ее в сокращенную форму строки-эшелона. Из этой формы мы можем интерпретировать решение системы уравнений.Чтобы поставить систему …

Матричные решатели (калькуляторы) с шагами. Вы можете использовать дроби, например, 1/3. Вычислить определитель, ранг и обратную матрицу … Решение системы из n линейных …

Введите матрицу, и этот калькулятор покажет вам шаг за шагом, как преобразовать эту матрицу в сокращенную форму эшелона строк с помощью Гаусса -Jordan Elmination.

Решите матричное уравнение (смесь) Линейные системы: запишите как матрицу Линейные системы: запишите как линейное уравнение Линейные системы: используйте обратную матрицу для решения Используйте данную обратную матрицу, чтобы решить для x, y и z Расширенные матрицы: Напишите расширенную матрицу. Расширенные матрицы: напишите расширенную матрицу и решите

Решающие системы уравнений с помощью rref: (Имея расширенную матрицу, вы можете уменьшить строку, используя следующие шаги калькулятора) 1.Нажмите 2ND, x 1 (MATRIX), прокрутите вправо до EDIT, нажмите ENTER 2. Введите размеры матрицы и введите расширенную матрицу (помните, что вы вводите правильные числа для матрицы) 3.

11 февраля 2015 · Система уравнений. Определение расширенной матрицы: Матрица, используемая для представления системы линейных уравнений, называется расширенной матрицей. Если — система уравнений, то расширенная матрица. Переставьте систему уравнений. Тогда расширенная матрица.Решение: расширенная матрица.

Система уравнений, сформированная с использованием сингулярной матрицы, имеет либо 0 решений, либо бесконечное число решений. Единственный способ определить, какой из них имеет место, — использовать расширенную матрицу и максимально упростить ее. В конечном итоге расширенная матрица даст строку следующего формата: 0 0 0… 0: c. где …

Этот калькулятор решает систему линейных уравнений любого вида с указанными шагами, используя либо метод исключения Гаусса-Жордана, либо метод Крамера.Чтобы решить любую систему, воспользуйтесь калькулятором системы уравнений. Показать инструкции.

4.2 Системы линейных уравнений и расширенных матриц 1. Решать более сложные линейные системы вручную нецелесообразно. 2. Компьютеры и калькуляторы теперь имеют встроенные процедуры для решения более крупных и сложных систем. Матрицы в сочетании с графическими утилитами и / или компьютерами используются для решения более сложных систем. 3.

Инструмент для вычисления изменения базовой матрицы на основе гомотетии или вращения в векторном пространстве и вычислений изменения координат.Как рассчитать замену базисных уравнений? Из матрицы преобразования $ P $ (также называемой базисной заменой базисной матрицы) любой вектор $ v $ становится …

Расширенная матрица: давайте рассмотрим систему уравнений. Расширенная матрица вышеупомянутой системы есть. Форма эшелона строк: матрица находится в форме эшелона строк, когда. Запись в строке 1, столбец 1 — 1, и 0 появляется под ней

28 февраля 2017 · Автор ejbarth Опубликовано 28 февраля 2017 4 марта 2017 Категории Линейная алгебра, Без категории Теги расширены, Ax = b, обратное решение, эшелон, линейная система, матричное уравнение 2 мысли о «Решении матричного векторного уравнения Ax = b в Maxima»

Матричный обратный калькулятор.Что такое системы уравнений? Система уравнений — это набор из одного или нескольких уравнений, включающих ряд переменных. Решениями систем уравнений являются отображения переменных, при которых удовлетворяются все составляющие уравнения — другими словами, местоположения в …

18 января 2008 г. · Я могу придумать как минимум четыре способа: 1) исключение (добавить или вычтите уравнения, чтобы исключить переменную) 2) подстановка (переставьте одно уравнение, чтобы получить одно уравнение само по себе, а затем подставьте в другое уравнение 3) Правило Крамера (матричные операции) 4) Графически (ищите пересечение) Да, система может есть более одного решения — если все строки являются одной и той же строкой, существует…

Если в системе нет решения, устраните проблемы. С помощью калькулятора найти A –1 * B проще простого. Вопрос: O СИСТЕМЫ УРАВНЕНИЙ И МАТРИЦ Решение несовместной системы линейных уравнений 2×2 или … Две системы уравнений приведены ниже. Затем вставьте формулу, показанную ниже. Для каждой системы выберите наилучшее описание ее решения. Затем система уравнений …

Используйте метод исключения Гаусса для решения системы уравнений, представленной в виде расширенной матрицы.Интерпретируйте решение системы уравнений, представленное в виде расширенной матрицы. Мы увидели, как написать систему уравнений с расширенной матрицей, а затем как использовать строковые операции и обратную подстановку для получения строчно-эшелонированной формы.

Private Sub cmdSolve_Click (ByVal eventSender As _ System.Object, ByVal eventArgs As System.EventArgs) _ Обрабатывает cmdSolve.Click Const TINY As Double = 0,00001 Dim num_rows As Integer Dim num_cols As Integer Dim tmp Asim Dimr factor (,) As Double Dim orig_arr (,) As Double Dim txt As String ‘Построить расширенную матрицу.arr …

d .. Какого размера матрица расширенных коэффициентов для системы из 3-х уравнений с 4-мя переменными? е. Коммутативно ли сложение матриц вообще? Калькулятор показывает расчет каждого элемента сопряженной матрицы.

Добавить комментарий

Ваш адрес email не будет опубликован. Обязательные поля помечены *